SlideShare a Scribd company logo
1 of 96
Download to read offline
According to the report of water quality control, drinking city water has the following characteristics:
turbidity - 1,5 mg/m3, odour - 3 points, metallic taste - 2 points, pale yellow colour, colour index -
20o, temperature - 12o. Which of these factors doesn't comply with hygienic requirements?
Select one:
a. Taste
b. Turbidity
c. Colour index
d. Odour
e. Temperature
Feedback
The correct answer is: Odour
Question 2
Not answered
Mark 0.00 out of 1.00
Flag question
Question text
The air of a foundry worker's working zone contains condensation aerosol with dust particles sized 2
nm (90%), 2-5 nm (2%), over 5 nm (6%), below 2 nm (about 2%). Characterize the dust dispersivity:
Select one:
a. Mist
b. Fine-dispersed
c. Median-dispersed
d. Coarsely dispersed
e. Ultrafine-dispersed
Feedback
The correct answer is: Fine-dispersed
Question 3
Not answered
Mark 0.00 out of 1.00
Flag question
Question text
A district doctor keeps the record of reconvalescents after infectious diseases, people who are
disposed to frequent and long-lasting diseases, patients with chronic pathologies. What category of
patients should belong to the III health group?
Select one:
a. People disposed to frequent and long-lasting diseases
b. Reconvalescents after infectious diseases and patients with chronic pathologies
c. People with chronic pathologies and disposed to frequent and long-lasting diseases
d. All above mentioned categories
e. People with chronic diseases
Feedback
The correct answer is: People with chronic diseases
Question 4
Not answered
Mark 0.00 out of 1.00
Flag question
Question text
To study physical development of children and adolescents, anthropometric studies are widely used.
Choose a physiometric method of study from the below given.
Select one:
a. Determination of vital capacity of lungs
b. Measurement of growth
c. Determination of vertebra form
d. Determination of thorax form
e. Determination of body weight
Feedback
The correct answer is: Determination of vital capacity of lungs
Question 5
Not answered
Mark 0.00 out of 1.00
Flag question
Question text
As a result of prophylactic medical examination a 35 year old woman was diagnosed with alimentary
and constitutive obesity of the III degree. It is known from her anamnesis that the patient doesn't
observe rules of rational nutrition: she often overeats, the last food intake is usually 10-15 minutes
before going to bed, prefers fattening and rich in carbohydrates food. What is the main alimentary
risk factor of obesity development?
Select one:
a. Excess of fats
b. Excess of carbohydrates
c. Energetic unprofitableness of nutrition
d. Violation of dietary pattern
e. Lack of cellulose
Feedback
The correct answer is: Energetic unprofitableness of nutrition
Question 6
Not answered
Mark 0.00 out of 1.00
Flag question
Question text
Some of the population of a city district have uneven teeth color. The individuals have white spots,
transverse brown stripes on the incisors. Occurrence of these symptoms is associated with the
quality of drinking water from a deep well. Which of the following components of water can be the
cause of the disease?
Select one:
a. Fe
b. Mg
c. J
d. Ca
e. F
Feedback
The correct answer is: F
Question 7
Not answered
Mark 0.00 out of 1.00
Flag question
Question text
The results of 5 year monitoring allowed to estimate the level of environmental influence upon health
indices of popultaion. What statistic method should be chosen?
Select one:
a. Calculation of regression coefficient
b. Calculation of coefficient of difference validity
c. Calculation of dynamic indices
d. Calculation of conformity coefficient
e. Calculation of correlation coefficient
Feedback
The correct answer is: Calculation of correlation coefficient
Question 8
Not answered
Mark 0.00 out of 1.00
Flag question
Question text
Researchers studied disease incidence of influenza and acute respiratory viral infection within the
last 5 years. What kind of graphic presentation should be used for the best visualization of this data?
Select one:
a. Linear diagram
b. Histogram
c. Radial diagram
d. Bar diagram
e. Pie diagram
Feedback
The correct answer is: Linear diagram
Question 9
Not answered
Mark 0.00 out of 1.00
Flag question
Question text
A student lives in the modern house in the flat with a complete set of sanitary equipment (WC, bath,
shower, local water heater). How much water consumption has he got?
Select one:
a. 500 -600 L/day
b. 300-400 L/day
c. 50-100 L/day
d. 160-200 L/day
e. 10-15 L/day
Feedback
The correct answer is: 160-200 L/day
Question 10
Not answered
Mark 0.00 out of 1.00
Flag question
Question text
A 30-year-old patient with complaints of occipital headache, disturbed sleep with nightmares came to
a policlinic. BP was 150/95 mm Hg. He was diagnosed with hypertensic crisis. The patient should be
registered in the following dispensary group for arterial hypertension surveillance:
Select one:
a. In the fifth
b. In the fourth
c. In the first
d. In the second
e. In the third
Feedback
The correct answer is: In the second
Question 11
Not answered
Mark 0.00 out of 1.00
Flag question
Question text
A young patient who came to a policlinic was diagnosed with the 1 stage of hypertension. How often
should he undergo the medical check-up?
Select one:
a. Twice a year
b. Once a year
c. 3 times a year
d. 5 times a year
e. 4 times a year
Feedback
The correct answer is: Twice a year
Question 12
Not answered
Mark 0.00 out of 1.00
Flag question
Question text
Study of morbidity rate in a city N revealed that population of different administrative districts differed
in age structure. What statistic method allows to eliminate influence of this factor upon morbidity
indices?
Select one:
a. Correlative regressive analysis
b. Wilcoxon's t-criterion
c. Analysis of dynamic series
d. Standardization
e. Calculation of average values
Feedback
The correct answer is: Standardization
Question 13
Not answered
Mark 0.00 out of 1.00
Flag question
Question text
A driver had been fixing a car in a closed garage and afterwards complained about headache,
dizziness, nausea, muscle asthenia, sleepiness. Objectively: pulse and respiratory rate elevation,
excitement, hypertension, delirium of persecution. What is the most likely diagnosis?
Select one:
a. Asthenovegetative syndrome
b. Hypertensive crisis
c. Intoxication with carbon oxide
d. Intoxication with ethyl gasoline
e. Posttraumatic encephalopathy
Feedback
The correct answer is: Intoxication with carbon oxide
Question 14
Not answered
Mark 0.00 out of 1.00
Flag question
Question text
A factory's sectorial doctor selects a group of persons who often fall ill for thorough monitoring. At
the same time he takes into consideration the number of etiologically related cases with temporary
disability in each of the employees over the last year. An employee falls into this group if the number
of sickness cases is:
Select one:
a. 3 or more
b. 2 or more
c. 6 or more
d. 1 or more
e. 4 or more
Feedback
The correct answer is: 4 or more
Question 15
Not answered
Mark 0.00 out of 1.00
Flag question
Question text
A city's population is 400000 inhabitants in 2005 there were registered 5600 deaths, including 3300
cases caused by cardiovascular diseases, 730 - by tumours. Which of the following indicators allows
to characterize the share of the circulatory system diseases as the cause of death in the city?
Select one:
a. Ratio
b. Intensive index
c. Demonstrativeness index
d. Index of relative intensity
e. Extensive index
Feedback
The correct answer is: Extensive index
Question 16
Not answered
Mark 0.00 out of 1.00
Flag question
Question text
An outpatient hospital made record of 11600 diseases within one year. Among them influenza and
ARD make up 5800, circulatory system diseases - 3480, digestion diseases - 1300, other diseases -
1020. What relative index can be calculated according this data?
Select one:
a. Visualization
b. Intensive
c. Extensive
d. Correlation
Feedback
The correct answer is: Extensive
Question 17
Not answered
Mark 0.00 out of 1.00
Flag question
Question text
A patient consulted a doctor about acure respiratory viral infection. The patient was acknowledged to
be off work. The doctor issued him a medical certificate for 5 days. The patient is not recovering.
What measures should the doctor take in order to legalize the further disability of patient?
Select one:
a. To prolong the medical certificate together with department superintendent
b. To send the patient to the medical consultative commission
c. To prolong the medical certificate at his own discretion but no more than for 6 days in total
d. To send the patient to the medical social expert comission
e. To prolong the medical certificate at his own discretion but no more than for 10 days in total
Feedback
The correct answer is: To prolong the medical certificate at his own discretion but no more than for
10 days in total
Question 18
Not answered
Mark 0.00 out of 1.00
Flag question
Question text
What information gathering method is preferable to study housing conditions of medical students
during training period?
Select one:
a. Materials selection
b. Directed selection method
c. Interviewing
d. Statistical
e. Questionaire
Feedback
The correct answer is: Questionaire
Question 19
Not answered
Mark 0.00 out of 1.00
Flag question
Question text
A pupil of the 8th form after trauma has acute atrophy of the left arm muscles, tonus of which is
distinctly decreased, active movements are only in the left joint, pupil's foot is deformed. Function of
support of the left leg is absent, support function of the right leg is preserved. The boy wears an
orthopedic footwear. What group of physical training does the boy belong to?
Select one:
a. Additional
b. Basic
c. Preparatory
d. Special
e. Other
Feedback
The correct answer is: Special
Question 20
Not answered
Mark 0.00 out of 1.00
Flag question
Question text
During the medical examination a port crane operator complained of dizziness, nausea, sense of
pressure against tympanic membranes, tremor, dyspnoea, cough. He works aloft, the work is
connected with emotional stress. Workers are affected by vibration (general and local), noise,
ultrasound, microclimate that warms in summer and cools in winter. What factor are the worker's
complaints connected with?
Select one:
a. Intensity of work
b. Altitude work
c. Vibration
d. Noise
e. Infrasound
Feedback
The correct answer is: Infrasound
Question 21
Not answered
Mark 0.00 out of 1.00
Flag question
Question text
In terms of megacalorie (1000 kcal = 4184 kJ) the ration of an adult includes 30 g of proteins, 37 g of
fats, 137 g of carbohydrates, 15 mg of vitamin C, 0,6 mg of thiamine (vitamin B1). The ration is
UNBALANCED as to the contents of:
Select one:
a. Fats
b. Proteins
c. Carbohydrates
d. Vitamin C
e. Thiamine
Feedback
The correct answer is: Vitamin C
Question 22
Not answered
Mark 0.00 out of 1.00
Flag question
Question text
Administration of a plant producing red lead paint intends to form a group of medical specialists for
periodical medical examinations. What specialist must be obligatory included into this group?
Select one:
a. Gynaecologist
b. Neuropathologist
c. Dermatologist
d. Psychiatrist
e. Otolaryngologyst
Feedback
The correct answer is: Neuropathologist
Question 23
Not answered
Mark 0.00 out of 1.00
Flag question
Question text
There were registered 500 cases of urolithiasis per 10000 inhabitants. What kind of statictical indices
is presented?
Select one:
a. Index of visualization
b. Index of compliance
c. Prevalence rate
d. Correlation coefficient
e. Incidence rate
Feedback
The correct answer is: Prevalence rate
Question 24
Not answered
Mark 0.00 out of 1.00
Flag question
Question text
Chief physician of a polyclinic encharged a district doctor with a task to determine the pathological
prevalence of disease N in his district. What document allows to estimate the disease prevalence in
the population of a medical district?
Select one:
a. Statistic coupons (+)
b. Statistic coupons (+) and (-)
c. Statistic coupons (-)
d. Prophylactic examinations register
e. Vouchers for medical appointments
Feedback
The correct answer is: Prophylactic examinations register
Question 25
Not answered
Mark 0.00 out of 1.00
Flag question
Question text
Workers of fishery are subjected to low temperatures of the air (from 5 till 150C). Diseases of what
organs and systems are the most frequent among workers of such enterprises?
Select one:
a. Respiratory system
b. Liver
c. Gastrointestinal tract
d. Cardiovascular system
e. Blood
Feedback
The correct answer is: Respiratory system
Question 26
Not answered
Mark 0.00 out of 1.00
Flag question
Question text
What juice should be included in a complex drug and dietary therapy for patients with gastric ulcer or
duodenal ulcer and increased gastric juice acidity in order to accelerate the ulcer healing?
Select one:
a. Cabbage, cabbage and carrot
b. Apple, birch and apple
c. Celery, parsley
d. Pumpkin
e. Potato, potato and carrot
Feedback
The correct answer is: Potato, potato and carrot
Question 27
Not answered
Mark 0.00 out of 1.00
Flag question
Question text
A patient complained about problems with pain and tactile sensitivity, pain in the nail bones at the
end of the working day. He works at a plant with mechanical devices. What pathology can be
suspected?
Select one:
a. Hypovitaminosis of B1
b. Caisson disease
c. Overwork symptoms
d. Vibration disease
e. Noise disease
Feedback
The correct answer is: Vibration disease
Question 28
Not answered
Mark 0.00 out of 1.00
Flag question
Question text
A children's health camp received a party of tinned food. External examination of the tins revealed
that they had deep dents, could be easily concaved when pressed and wouldn't immediately return
to the initial state; rust was absent; the tins were greased with inedible fat. Specify the bloat type:
Select one:
a. Combined
b. Chemical
c. Physical
d. Physicochemical
e. Biological
Feedback
The correct answer is: Physical
Question 29
Not answered
Mark 0.00 out of 1.00
Flag question
Question text
Preventive examination of an 11 year old boy helped to determine his habitus type. It was
established that the child's shoulders were deviated and brought forward, with forward flexion of
head, the thorax was flattened, abdomen was convex. The child's backbone had signs of deepened
cervical and lumbar curvatures. What habitus is it?
Select one:
a. Lordosis
b. Normal
c. Round-shouldered
d. Corrected
e. Kyphosis
Feedback
The correct answer is: Kyphosis
Question 30
Not answered
Mark 0.00 out of 1.00
Flag question
Question text
The total area of a ward at the therapeutical department is 28 m2. What is the maximum number of
beds that can be exploited in this ward?
Select one:
a. 5
b. 2
c. 4
d. 1
e. 3
Feedback
The correct answer is: 4
Question 31
Not answered
Mark 0.00 out of 1.00
Flag question
Question text
Working conditions of a building company worker are characterized by cooling microclimate effect,
silica-containing dust, caustic alkali (quicklime) and noise. What medical expert should be the chief
of the commission that periodically examines the workers of the mentioned category?
Select one:
a. Neurologist
b. Therapeutist
c. Ophthalmologist
d. Dermatologist
e. Otolaryngologist
Feedback
The correct answer is: Therapeutist
Question 32
Not answered
Mark 0.00 out of 1.00
Flag question
Question text
In order to improve organism tolerance of boarding-school pupils a doctor developed a program. The
program is based upon the following principles: graduality, consistency, individuality, coomplexity.
What of the main principles of organism tempering wasn't taken into account?
Select one:
a. Increase of resistance
b. Increase of influence intensity
c. Increase of influence force
d. Systematicness
e. Autodefense increase
Feedback
The correct answer is: Systematicness
Question 33
Not answered
Mark 0.00 out of 1.00
Flag question
Question text
At year-end hospital administration has obtained the following data: annual number of treated
patients and average annual number of beds used for patients' treatment. What index of hospital
work can be calculated on the base of this data?
Select one:
a. Average annual bed occupancy
b. Bed resources of the hospital
c. Average duration of patients' presence in the hospital
d. Average bed idle time
e. Bed turnover
Feedback
The correct answer is: Bed turnover
Question 34
Not answered
Mark 0.00 out of 1.00
Flag question
Question text
On medical observation a doctor identified girl (162 cm tall and 59 kg weight) who complained loss
of ability to see surrounding objects clearly in the evening. On examination: dry skin, hyperkeratosis.
Her daily ration includes the following vitamines: vitamine А– 0,5 mg, vit.В1– 2,0 mg, vit.В2– 2,5 mg,
vit.В6– 2 mg, vit.С– 70 mg. What is the hypovitaminosis type?
Select one:
a. B2-hypovitaminosis
b. B6-hypovitaminosis
c. A-hypovitaminosis
d. B1-hypovitaminosis
e. C-hypovitaminosis
Feedback
The correct answer is: A-hypovitaminosis
Question 35
Not answered
Mark 0.00 out of 1.00
Flag question
Question text
200 patients suffering from essential hypertension were examined in order to obtain data about
patients' arterial pressure and age. What statistic value should be applied in order to measure
relation between these characteristics?
Select one:
a. Student's coefficient
b. Correlation coefficient
c. Representation error
d. Sygmal deviation
e. Coefficient of variation
Feedback
The correct answer is: Correlation coefficient
Question 36
Not answered
Mark 0.00 out of 1.00
Flag question
Question text
The major repair of a hospital included renewal of colour design of hospital premises because it is of
great psychological and aesthetical importance; and so the walls of patient wards will be painted
under consideration of:
Select one:
a. Wall reflection coefficient
b. Hospital profile
c. Creation of cozy atmosphere
d. Windows orientation
e. Diseases of patients who will be staying in these wards
Feedback
The correct answer is: Windows orientation
Question 37
Not answered
Mark 0.00 out of 1.00
Flag question
Question text
In a city with population 400000 people 5600 fatal cases were recorded, including 3300 cases
because of blood circulation diseases, 730 - because of tumors. What index will allow to
characterize mortality from blood circulation diseases in this city?
Select one:
a. Relative intensity index
b. Visuality index
c. Correlation index
d. Extensive index
e. Intensive index
Feedback
The correct answer is: Intensive index
Question 38
Not answered
Mark 0.00 out of 1.00
Flag question
Question text
Choose a method of a graphic representation of monthly information about number of the registered
cases of acute intestinal infection and their comparisons to the average monthly values, obtained for
5 previous years:
Select one:
a. The figured diagram
b. The curvilinear disgram
c. The radial diagram
d. The linear diagram
e. The sector diagram
Feedback
The correct answer is: The linear diagram
Question 39
Not answered
Mark 0.00 out of 1.00
Flag question
Question text
A 38 year old man was admitted to a hospital from his working place on July 19 because of hip
fracture. He was invalid till November 19. Requires prolongation of treatment. Who decides on the
issue of further temporary invalidity?
Select one:
a. DCC
b. Regional MSEC
c. The head physician of a polyclinic
d. Specialized (traumatologic) MSEC
e. Interregional general MSEC
Feedback
The correct answer is: Specialized (traumatologic) MSEC
Question 40
Not answered
Mark 0.00 out of 1.00
Flag question
Question text
A patient who has been consuming refined foodstuffs for a long time complains about headache,
fatiguability, depression, insomnia, irritability. Objectively: muscle asthenia, pain and cramps in the
gastrocnemius muscles, during walking the patient lands onto his heel first, then on the external
edge of foot. Cardiovascular system exhibits tachycardia, hypoxia, dystrophic changes of
myocardium. There are also gastrointestinal disorders. What is the most likely diagnosis?
Select one:
a. Hypovitaminosis B6
b. Hypovitaminosis B12
c. Hypovitaminosis B15
d. Hypovitaminosis B2
e. Hypovitaminosis B1
Feedback
The correct answer is: Hypovitaminosis B1
Question 41
Not answered
Mark 0.00 out of 1.00
Flag question
Question text
Deputy of chief medical officer carried out a study of morbidity rate for population which had been
served at the polyclinics within the last 5 years. What statistical values can help in calculation of
morbidity rates?
Select one:
a. Average values
b. Dynamic series
c. Absolute values
d. Relative values
e. Standard values
Feedback
The correct answer is: Relative values
Question 42
Not answered
Mark 0.00 out of 1.00
Flag question
Question text
A selective population research study was aimed at exploring the effect of air emissions from a
metallurgical plant on the obstructive bronchitis morbidity in a city. The calculated correlation
coefficient was +0,79. Evaluate the strength and direction of the relationship:
Select one:
a. Inverse, strong
b. Inverse, average
c. Direct, strong
d. Direct, average
Feedback
The correct answer is: Direct, strong
Question 43
Not answered
Mark 0.00 out of 1.00
Flag question
Question text
At first appointment with an obstetrician-gynaecologist a pregnant woman is referred to other
medical specialists. She must be obligatory examined by the following specialists:
Select one:
a. Dentist and phthisiatrician
b. ENT and ophthalmologist
c. Therapeutist and dentist
d. Therapeutist and endocrinologist
e. Dentist and cardiologist
Feedback
The correct answer is: Therapeutist and dentist
Question 44
Not answered
Mark 0.00 out of 1.00
Flag question
Question text
During coal extraction in a mine the concentration of coal dust in the working area is 450 mg/m3
(MPC is 10 mg/m3). What occupational respiratory disease may develop in miners?
Select one:
a. Anthracosis
b. Allergic nasopharyngitis
c. Talcosis
d. Siderosis
e. Byssinosis
Feedback
The correct answer is: Anthracosis
Question 45
Not answered
Mark 0.00 out of 1.00
Flag question
Question text
Over a current year among workers of an institution 10% haven't been ill a single time, 30% have
been ill once, 15% - twice, 5% - 4 times, the rest - 5 and more times. What is the percentage of
workers relating to the I health group?
Select one:
a. 60%
b. 10%
c. 22%
d. 55%
e. 40%
Feedback
The correct answer is: 55%
Question 46
Not answered
Mark 0.00 out of 1.00
Flag question
Question text
A military unit stopped for 3-day's rest in an inhabited locality after a long march. The sanitary-
epidemiological reconnaissance found several water sources. It is necessary to choose the source
complying with the hygienic standards for drinking water in the field conditions:
Select one:
a. Melt snow water
b. Artesian well water
c. River water
d. Spring water
e. Rain water
Feedback
The correct answer is: Artesian well water
Question 47
Not answered
Mark 0.00 out of 1.00
Flag question
Question text
Analysis of organization of medical care in a regional centre has shown that every year about 12% of
patients receive inpatient care for diseases that don't require round-the-clock monitoring and
intensive care. What are the most appropriate organizational changes required to address this
problem?
Select one:
a. Development of medical care forms replacing the in-patient care
b. Development of primary care
c. Changes to the statute of outpatient clinics
d. Upgrading of hospital facilities
e. Restructuring of specialized care
Feedback
The correct answer is: Development of medical care forms replacing the in-patient care
Question 48
Not answered
Mark 0.00 out of 1.00
Flag question
Question text
A heat station working on solid fuel is located in a residential district. On cloudy foggy days in
december there was an increase in diseases with upper airway affection and signs of general
intoxication. There were also mortal cases among the elderly people. What is the most likely factor
that provoked toxic effect?
Select one:
a. Suspended materials
b. Low air temperature
c. High air humidity
d. Temperature gradient
e. Calm
Feedback
The correct answer is: Suspended materials
Question 49
Not answered
Mark 0.00 out of 1.00
Flag question
Question text
District physician was charged with plan drafting concerning medical and preventive measures
among the population in the area he is assigned to. What measures must he include in this plan as
regards primary prevention of illness?
Select one:
a. Measures to improve patients' life conditions
b. Prevention of disease onset
c. Measures to increase patients' life quality
d. Prevention of disease complications
e. Referral of patients to sanatorium
Feedback
The correct answer is: Prevention of disease onset
Question 50
Not answered
Mark 0.00 out of 1.00
Flag question
Question text
A worker diagnosed with "acute dysentery" was sent to the infectious department by a doctor of aid
post. What document should be used for registration of this disease?
Select one:
a. Outpatient's card
b. Statistic card of the patient who left in-patient hospital
c. Statistic coupon for registration of final diagnoses
d. Urgent report on infectious disease
e. Inpatient's card
Feedback
The correct answer is: Urgent report on infectious disease
Question 51
Not answered
Mark 0.00 out of 1.00
Flag question
Question text
In one of the surgical departments the quality assurance testing of sterilization of surgical
instruments was performed. After an instrument had been treated with 1% phenolphthalein, the
solution turned pink. This indicates that the instrument has:
Select one:
a. Residual blood
b. Drugs residues
c. Residual tissue
d. Synthetic detergent residues
e. Disinfectant residues
Feedback
The correct answer is: Synthetic detergent residues
Question 52
Not answered
Mark 0.00 out of 1.00
Flag question
Question text
A department chief of an in-patient hospital is going to inspect resident doctors as to observation of
medical-technological standards of patient service. What documentation should be checked for this
purpose?
Select one:
a. Annual report of a patient care institution
b. Registry of operative interventions
c. Health cards of in-patients
d. Treatment sheets
e. Statistic cards of discharged patients
Feedback
The correct answer is: Health cards of in-patients
Question 53
Not answered
Mark 0.00 out of 1.00
Flag question
Question text
While making sanitary examination of burn unit for adults it was stated that wards for 4 persons are
of 28 m2 square. What should be the minimum ward area in this unit?
Select one:
a. 52 м2
b. 28 м2
c. 30 м2
d. 40 м2
e. 24 м2
Feedback
The correct answer is: 40 м2
Question 54
Not answered
Mark 0.00 out of 1.00
Flag question
Question text
A plot of land with total area of 2,0 hectare was intended for building of a hospital. The maximal
capacity of the hospital will be:
Select one:
a. 200 beds
b. Over 1000 beds
c. 100 beds
d. 400 beds
e. 800 beds
Feedback
The correct answer is: 100 beds
Question 55
Not answered
Mark 0.00 out of 1.00
Flag question
Question text
A 59-year-old male patient with essential hypertension of stage II is registered with the dispensary
department of a polyclinic. The patient regularly takes ACE inhibitors and calcium antagonists. How
often should a therapeutist examine this patient (except for exacerbation periods)?
Select one:
a. Once a year
b. Every 6 months
c. Every 4 months
d. Every 9 months
e. Every 3 months
Feedback
The correct answer is: Every 3 months
Question 56
Not answered
Mark 0.00 out of 1.00
Flag question
Question text
A local doctor has to prepare a report about the health condition of the population of his region. What
medical indexes of population health condition should he use?
Select one:
a. Average treatment duration, complications
b. Average longevity
c. Morbidity, disabilities, demographic, physical development
d. Social welfare, satisfaction of life quality
e. Way of life, genetic, pollution
Feedback
The correct answer is: Morbidity, disabilities, demographic, physical development
Question 57
Not answered
Mark 0.00 out of 1.00
Flag question
Question text
Within the structure of the region's population the share of persons aged 0 to 14 years is 25%, the
share of persons aged 50 years and older is 30%. What concept most accurately describes this
demographic situation?
Select one:
a. Cohort reproduction
b. Progressive type of population age structure
c. Regressive type of population age structure
d. Immigration of population
e. Stationary type of population age structure
Feedback
The correct answer is: Regressive type of population age structure
Question 58
Not answered
Mark 0.00 out of 1.00
Flag question
Question text
A 2 year old child has been ill with acute respiratory viral infection of upper thrice a year - in
February, in April and in December. How should these occurences be recorded?
Select one:
a. It is necessary to fill in 1 statistic talon signed (-)
b. It is necessary to fill in 1 statistic talon signed (+) and 2 statistic talons signed (-)
c. It is necessary to fill in 3 statistic talons signed (+)
d. It is necessary to fill in 1 statistic talon signed (+)
e. It is necessary to fill in 3 statistic talons signed (-)
Feedback
The correct answer is: It is necessary to fill in 3 statistic talons signed (+)
Question 59
Not answered
Mark 0.00 out of 1.00
Flag question
Question text
In treatment and prevention establishments, regardless of their organisational and proprietary form,
the rights of patients should be observed. Which of these rights is the most significant?
Select one:
a. The right to the protection of the patient's interests
b. The right to the free choice
c. The right to the information
d. The right to be heard
e. The right to the protection from incompetence
Feedback
The correct answer is: The right to the protection of the patient's interests
Question 60
Not answered
Mark 0.00 out of 1.00
Flag question
Question text
A factory worker has ARD complicated by acute bronchitis. He receives treatment in the outpatient
setting. The attending doctor has issued him a medical certificate for 5 days and then extended its
duration by 5 more days. Patient can't get down to work because of his health status. Who should
extend the duration of medical certificate for this patient?
Select one:
a. A department chief
b. Deputy medical superintendent in charge of medical treatment
c. Medical advisory commission
d. Medical superintendent
e. Deputy medical superintendent in charge of temporary disability examination
Feedback
The correct answer is: A department chief
Question 61
Not answered
Mark 0.00 out of 1.00
Flag question
Question text
Point out the unit for statistical observation for the determination of blood sugar level influence on a
wound surface healing during postoperative period.
Select one:
a. The patient who has a wound surface
b. The patient in a postoperative period
c. Blood test
d. Blood sugar level
e. The patient who was discharged to outpatient treatment
Feedback
The correct answer is: The patient in a postoperative period
Question 62
Not answered
Mark 0.00 out of 1.00
Flag question
Question text
A district pediarician has carried out infant mortality rate analysis in his area. What data has been
used?
Select one:
a. Mortality of children under 1 y.o., natimortality
b. Mortality of children under 1 y.o. structured by age, sex, causes
c. Mortality of district newborn
d. Hospital mortality of children, structured by age
e. Mortality of district adolescents
Feedback
The correct answer is: Mortality of children under 1 y.o. structured by age, sex, causes
Question 63
Not answered
Mark 0.00 out of 1.00
Flag question
Question text
The Carpathian region is characterized by constant high humidity of atmospheric air (over 80%).
Inhabitants of this region feel severe cold in corresponding season at a medium low temperature. It's
caused by heat emission by:
Select one:
a. Convection
b. Conduction
c. Vaporization
d. Radiation
Feedback
The correct answer is: Convection
Question 64
Not answered
Mark 0.00 out of 1.00
Flag question
Question text
A district doctor was commisioned with a task to work out a plan of treatment-and-prophylaxis
actions for the population of his district. What actions of secendary prophylaxis must he include into
this plan?
Select one:
a. Rehabilitation actions
b. Prevention of disease complications
c. Elimination of disease causes
d. Improvement of population's living conditions
e. Disease prevention
Feedback
The correct answer is: Prevention of disease complications
Question 65
Not answered
Mark 0.00 out of 1.00
Flag question
Question text
During the periodic medical examination an assembly fitter (works on soldering details) didn't report
any health problems. Closer examination revealed signs of asthenic-vegetative syndrome. Blood
included red blood cells with basophilic aggregations and a somewhat higher number of
reticulocytes, urine had a high concentration of delta-aminolevulinic acid. The complex of symptoms
indicates the initial stage of chronic intoxication with:
Select one:
a. Ethanol
b. Lead
c. Mercury
d. Manganese
e. Tin
Feedback
The correct answer is: Lead
Question 66
Not answered
Mark 0.00 out of 1.00
Flag question
Question text
An outbreak of food poisoning was recorded in an urban settlement. The illness was diagnosed as
botulism on the grounds of clinical presentations. What foodstuffs should be chosen for analysis in
the first place in order to confirm the diagnosis?
Select one:
a. Pasteurized milk
b. Potatoes
c. Cabbage
d. Boiled meat
e. Tinned food
Feedback
The correct answer is: Tinned food
Question 67
Not answered
Mark 0.00 out of 1.00
Flag question
Question text
Environmental pollution is prevented by mechanical separation of nontoxic solid domestic waste.
Specify the method which can be used for mechanical utilization of these wastes:
Select one:
a. Burning as power-plant fuel
b. Waste neutralization in biothermal boxes
c. Compressing of wastes into building blocks
d. Burial of wastes
e. Hydrolysis
Feedback
The correct answer is: Compressing of wastes into building blocks
Question 68
Not answered
Mark 0.00 out of 1.00
Flag question
Question text
A 50-year-old male suburbanite underwent treatment in rural outpatient clinic for pneumonia. The
treatment didn't have effect and the disease got complicated by exudative pleuritis. What prevention
and treatment facility should the patient be referred to for further aid?
Select one:
a. Tuberculosis dispensary
b. Regional hospital
c. Central district hospital
d. Phthisio-pulmonological dispensary
e. Municipal hospital
Feedback
The correct answer is: Central district hospital
Question 69
Not answered
Mark 0.00 out of 1.00
Flag question
Question text
There is a dynamic growth of number of congenital abnormalities such as central paralysis,
newborns blindness, idiocy among the population that lives near to pesticides production enterprise.
Compounds of which pollutant can cause the development of this pathology?
Select one:
a. Strontium
b. Chrome
c. Cadmium
d. Mercury
e. Iron
Feedback
The correct answer is: Mercury
Question 70
Not answered
Mark 0.00 out of 1.00
Flag question
Question text
A mother who is on partially paid maternity leave got sick and was hospitalized. What document is to
be issued to a working father who will be taking care of a child during his mother's illness?
Select one:
a. Certificate of mother's illness
b. Free-form certificate
c. Certificate of child's care necessity
d. Sick-leave
e. Extract from the medical card of out- or in-patient
Feedback
The correct answer is: Sick-leave
Question 71
Not answered
Mark 0.00 out of 1.00
Flag question
Question text
Poorly refined wastes of an industrial plant are usually thrown into the river that supplies drinking
water. It causes perishing of some microorganisms, disturbs processes of water self-purification and
worsens its quality that can have negative influence upon people's health. How is this effect of
environmental factors called?
Select one:
a. Combined
b. Associated
c. Indirect
d. Complex
e. Direct
Feedback
The correct answer is: Indirect
Question 72
Not answered
Mark 0.00 out of 1.00
Flag question
Question text
District doctor of rural medical department was called to a 42-year-old patient. While examining the
patient, the doctor suspected the dysentery. What document must the doctor issue?
Select one:
a. Report addressed to Head of the village
b. Statistical coupon of final (precise) diagnosis
c. Urgent notification of infectious disease
d. Abstract of outpatient medical card
e. Infectious disease report
Feedback
The correct answer is: Urgent notification of infectious disease
Question 73
Not answered
Mark 0.00 out of 1.00
Flag question
Question text
The institutions which take part in medical examinations can be prevention and treatment facilities,
medical board of Ministry of Defense, medical board of Ministry of Home Affairs, medico-social
expert commissions, forensic medical boards etc. What institutions are responsible for temporary
disability examination?
Select one:
a. Medical boards of Ministry of Home Affairs
b. Sanitary-and-prophylactic institutions
c. Medico-social expert commissions
d. Medical boards of Ministry of Defense
e. Prevention and treatment facilities
Feedback
The correct answer is: Prevention and treatment facilities
Question 74
Not answered
Mark 0.00 out of 1.00
Flag question
Question text
Periodical survey of a worker of a chemicals plant revealed a malignant neoplasm on the urinary
bladder. This occupational disease was the most probably caused by contact with the following
industrial poison:
Select one:
a. Benzidine
b. Vinyl chloride
c. Nickel carbonyl
d. Arsenic
e. Asbestos
Feedback
The correct answer is: Benzidine
Question 75
Not answered
Mark 0.00 out of 1.00
Flag question
Question text
The student has the following devices: Geiger counter, Ebert counter, Krotov's apparatus, Mischuk
device, Ebert device. What device can he use to assess air germ pollution?
Select one:
a. Mischuk's device
b. Geiger's counter
c. Krotov's apparatus
d. Ebert's device
e. Ebert's counter
Feedback
The correct answer is: Krotov's apparatus
Question 76
Not answered
Mark 0.00 out of 1.00
Flag question
Question text
Head of a department and a trade-union group have appealed to the head of a hospital about
dismissal of the senior nurse who has 17-year record of service. The facts of charge were confirmed
and recognized by the nurse herself. The nurse lives with a daughter (who is divorced and
unemployed) and a 9-month-old grandson. Make an administrative decision:
Select one:
a. To issue the sick leave
b. To discharge the worker, i.e. to satisfy demands of the collective
c. To continue the worker in office with a warning of dismissal in case of repeated violation of labor
discipline
d. To embark other officials or public organizations with this problem
Feedback
The correct answer is: To continue the worker in office with a warning of dismissal in case of
repeated violation of labor discipline
Question 77
Not answered
Mark 0.00 out of 1.00
Flag question
Question text
During the medical examination at school the schoolchildren had to undergo plantography. After the
analysis of footprints platypodia was found in 30% of pupils. What is the percentage of the flatfoot
isthmus?
Select one:
a. 30%
b. 45%
c. 65%
d. 55%
e. 50%
Feedback
The correct answer is: 65%
Question 78
Not answered
Mark 0.00 out of 1.00
Flag question
Question text
A patient undergoes inpatient treatment with the diagnosis of acute pancreatitis. To spare pancreas
as much as possible the doctor prescribed for him starvation for 1-3 days. What products is the
patient allowed to eat during recovery period after cancelling of starvation?
Select one:
a. Potato and carrot mash
b. Grape juice
c. Milk
d. Broth
e. Boiled meat
Feedback
The correct answer is: Potato and carrot mash
Question 79
Not answered
Mark 0.00 out of 1.00
Flag question
Question text
A 48-year-old male in-patient undergoes treatment for essential hypertension of II-B stage. It is
known from history that he works in a design engineering office. His job involves neuro-emotional
stress. Which of these foodstuffs do not stimulate the central nervous system and can be
recommended for the patient?
Select one:
a. Whole milk
b. Carbonated beverages
c. Meat broths
d. Vegetable broths
e. Mushroom broths
Feedback
The correct answer is: Whole milk
Question 80
Not answered
Mark 0.00 out of 1.00
Flag question
Question text
The amount of ultraviolet radiation dose was measured in minutes. What device was applied for
measurement of the biodose?
Select one:
a. UV-meter
b. Radiometer
c. Actinometer
d. Catathermometer
e. Gorbachev's biodosimeter
Feedback
The correct answer is: Gorbachev's biodosimeter
Question 81
Not answered
Mark 0.00 out of 1.00
Flag question
Question text
Among the inhabitants of a workmen's settlement located near an industrial plant the cases of
nervous and endocrine system diseases as well as renal diseases became more frequent. Blood
analyses revealed a decrease in sulfhydryl groups. These pathologies might have been caused by
the following substance released into the environment:
Select one:
a. Chromium
b. Mercury
c. Cadmium
d. Boron
e. Lead
Feedback
The correct answer is: Mercury
Question 82
Not answered
Mark 0.00 out of 1.00
Flag question
Question text
An employee has been diseased for 4 months, further treatment is necessary, the patient is
incapacitated. Who is authorized to provide further disability examination of this patient?
Select one:
a. Deputy chief responsible for disability examination
b. Medical and social expert board
c. Physician in charge and chief of department
d. Chief physician of a medical facility
e. Medical consultative board
Feedback
The correct answer is: Medical and social expert board
Question 83
Not answered
Mark 0.00 out of 1.00
Flag question
Question text
Statistic of patients of common medical practice displays constant increase in elderly and old people
number. What kind of pathology is expected to prevail in the morbidity structure of population in
question?
Select one:
a. Acute pathology
b. Chronic pathology
c. Non-epidemic pathology
d. Occupational pathology
e. Infectious pathology
Feedback
The correct answer is: Chronic pathology
Question 84
Not answered
Mark 0.00 out of 1.00
Flag question
Question text
What is the maximum duration of medical certificate in case of tuberculosis?
Select one:
a. 10 months
b. 2 months
c. Week
d. 2 weeks
e. Month
Feedback
The correct answer is: 2 months
Question 85
Not answered
Mark 0.00 out of 1.00
Flag question
Question text
It is planned to organize a rural outpatient clinic. The patients will be able to visit the doctors of the
following specialities:
Select one:
a. Obstetrician-gynaecologist, therapeutist
b. Therapeutist, dentist, pediatrician, obstetrician-gynecologist
c. Pediatrician, obstetrician-gynaecologist, ophthalmologist
d. Pediatrician, therapeutist, ophthalmologist
e. Therapeutist, pediatrician, neurologist
Feedback
The correct answer is: Therapeutist, dentist, pediatrician, obstetrician-gynecologist
Question 86
Not answered
Mark 0.00 out of 1.00
Flag question
Question text
A child is 6 years old. Within one year of observation he had URI that lasted 8 days. Physical state is
satisfactory. Specify hi health group:
Select one:
a. II
b. III (b)
c. III (c)
d. I
e. III (a)
Feedback
The correct answer is: I
Question 87
Not answered
Mark 0.00 out of 1.00
Flag question
Question text
A municipal hospital reported on the number of operated patients including fatal outcomes following
the operations. Which index of hospital work can be calculated on the ground of this data?
Select one:
a. Postoperative lethality
b. Index of late hospitalization since a disease incursion
c. Standardized lethality
d. Total lethality
Feedback
The correct answer is: Postoperative lethality
Question 88
Not answered
Mark 0.00 out of 1.00
Flag question
Question text
Head of a department and a trade-union group have appealed to the head of a hospital about
dismissal of the senior nurse who has 17 year record of service. The facts of charge were confirmed
and recognized by the nurse herself. This nurse lives with a daughter (who is divorced and
unemployed) and a 9-month-old grandson. Make an administrative decision:
Select one:
a. To issue the sick list
b. To shift the solution of this problem on other officials or public organizations
c. To discharge the worker, i.e. to satisfy demands of the collective
d. To continue the worker in office with a warning of dismissal in case of repeated violation of labor
discipline
Feedback
The correct answer is: To continue the worker in office with a warning of dismissal in case of
repeated violation of labor discipline
Question 89
Not answered
Mark 0.00 out of 1.00
Flag question
Question text
A 46-year-old patient once took part in elimination of breakdown at an atomic power plant. Currently
he is being treated at an in-patient hospital. He was diagnosed with progressing vegetative
insufficiency. This disease relates to the following group of ionizing radiation effects:
Select one:
a. Hormesis
b. Somato-stochastic
c. Heterosis
d. Somatic
e. Genetic
Feedback
The correct answer is: Somato-stochastic
Question 90
Not answered
Mark 0.00 out of 1.00
Flag question
Question text
Bacterial analysis of air in a living space in winter period by means of Krotov's apparatus revealed
that total number of microorganisms in 1 m3 of air was 7200. What is the permissible number of
microorganisms for the air to be characterized as "pure"?
Select one:
a. Up to 7500
b. Up to 5500
c. Up to 2500
d. Up to 4500
e. Up to 3500
Feedback
The correct answer is: Up to 4500
Question 91
Not answered
Mark 0.00 out of 1.00
Flag question
Question text
It is planned to make complete isolation boxes in the infectious department in order to prevent
nosocomial airborne infections. The boxes consist of a tambour, a ward and a lock chamber. What
structure should be also included in a complete isolation box?
Select one:
a. Doctor's consulting room
b. Patient's examination room
c. Manipulation room
d. Nursing room
e. Bathroom unit
Feedback
The correct answer is: Bathroom unit
Question 92
Not answered
Mark 0.00 out of 1.00
Flag question
Question text
An employee of a private company was ill with acute respiratory viral infection. Consulted a district
doctor, who determined the fact of temporary loss of working ability, but refused to issue a sick-list,
arguing that the patient worked in the private and not state-owned company. Should the sick-list be
issued to the employees of private companies?
Select one:
a. Issued only on condition of payment guarantee by the company's proprietor
b. Issued a medical certificate of a set form
c. Issued a medical certificate of a free form
d. Issued only to empties of state-owned companies
e. Issued regardless of company's ownership
Feedback
The correct answer is: Issued regardless of company's ownership
Question 93
Not answered
Mark 0.00 out of 1.00
Flag question
Question text
In a rural health district a child died in the first month of life. In order to analyze this situation it was
necessary to complete expert evaluation of medical records. Which medical document was analyzed
in the first place?
Select one:
a. Neonatal record
b. Medical record of an outpatient
c. Child developmental history record
d. Child's medical record
e. Record of vaccinations
Feedback
The correct answer is: Child developmental history record
Question 94
Not answered
Mark 0.00 out of 1.00
Flag question
Question text
Examination of a 13-year-old boy reveals that his body length is 147 сm (+2), body weight - 38 kg
(+1,5), circumference of chest - 72 cm (+0,2). Estimate the harmonicity of the child's physical
development:
Select one:
a. Disharmonious
b. Above the average
c. Sharply disharmonious
d. Harmonious
e. Supernormal
Feedback
The correct answer is: Disharmonious
Question 95
Not answered
Mark 0.00 out of 1.00
Flag question
Question text
Study of actual diet of an adult revealed the following: proteins make up 16% of energy value of daily
ration, fats - 25%, carbohydrates - 59%. Evaluate compliance of protein, fat and carbohydrate share
in the energy value of daily ration with the recommended shares of these nutrients?
Select one:
a. Carbohydrate share is insufficicent
b. Carbohydrate share is excessive
c. Fat share is insufficient
d. Nutrient content complies with the recommended shares of energy value
e. Carbohydrate share is insufficient, there is excess of proteins
Feedback
The correct answer is: Carbohydrate share is insufficient, there is excess of proteins
Question 96
Not answered
Mark 0.00 out of 1.00
Flag question
Question text
Atmospheric air of an industrial centre is polluted with the following wastes of metallurgical plants:
sulphuric, nitric, metal, carbon oxides that have negative influence upon the inhabitants' health. The
effct of these hazards can be characterized as:
Select one:
a. Mixed
b. Complex
c. Adjacent
d. Associated
e. Combined
Feedback
The correct answer is: Combined
Question 97
Not answered
Mark 0.00 out of 1.00
Flag question
Question text
A 37 year old patient applied to a local therapeutist. As a result of exacerbation of chronic
obstructive bronchitis the patient had been temporarily disabled for 117 days within 1 year. What
tactics will be legally correct?
Select one:
a. The therapeutist should extend a medical certificate
b. The patient shoul be referred to the sanatorium-and-spa treatment
c. The therapeutist should issue a new medical certificate
d. The patient should be referred to the medical consultation comission for extension of medical
certificate
e. The patient should be referred to the medicosocial expertise
Feedback
The correct answer is: The patient should be referred to the medicosocial expertise
Question 98
Not answered
Mark 0.00 out of 1.00
Flag question
Question text
350 workers of a metalurgical plant had to undergo a yearly preventive examination. A territorial
polyclinic carried out preventive examination of 325 workers. As a result of it, 1 worker was
recognized as temporarily disabled, 15 workers underwent further rehabilitation at an after-work
sanatorium, 10 workers were provided with diet meal. What index characterizing the preventive work
of the polyclinic should be applied in this case?
Select one:
a. Frequency of case detection during examinations
b. Coverage of preventive medical examinations
c. Percentage of people who underwent rehabilitation at an after-work sanatorium
d. Percentage of people who were provided with diet meal
e. Percentage of temporarily disabled people
Feedback
The correct answer is: Coverage of preventive medical examinations
Question 99
Not answered
Mark 0.00 out of 1.00
Flag question
Question text
What method is applied to establish rate of correlation between age of men and their mortality due to
myocardial infarction?
Select one:
a. The quadrate method (Pirson)
b. Correlation ratio
c. The Indirect method (Student)
d. Method of graduated correlation (Armler)
e. Method of grade correlation (Spirman)
Feedback
The correct answer is: Method of grade correlation (Spirman)
Question 100
Not answered
Mark 0.00 out of 1.00
Flag question
Question text
During inspection of sanitary conditions of studying at a technical university it was necessary to
evaluate the visual regimen of students, who study from 9 a.m to 3 p.m. What index of natural light
will be the most informative?
Select one:
a. Time of the room insolation
b. Presence of mixed (superolateral) light
c. Natural light coefficient
d. Depth of study room
e. Light coefficient
Feedback
The correct answer is: Natural light coefficient
Question 101
Not answered
Mark 0.00 out of 1.00
Flag question
Question text
The head of prenatal care department intends to complete expert evaluation of compliance with
medical and technological standards of pregnancy follow-up. What documents must be verified for
this purpose?
Select one:
a. Labour and delivery record
b. Neonatal record
c. Medical records of outpatients
d. Individual records of pregnant and postpartum women
e. Prenatal records
Feedback
The correct answer is: Individual records of pregnant and postpartum women
Question 102
Not answered
Mark 0.00 out of 1.00
Flag question
Question text
Studying of pulmonary tuberculosis incidence provided data about patients' socioeconomic living
conditions and bad habits. What method allows to estimate the impact of these factors on
tuberculosis incidence?
Select one:
a. Calculation of correspondence index
b. Calculation of regression coefficient
c. Standardized index calculation
d. Calculation of reliability coefficient
e. Calculation of correlation coefficient
Feedback
The correct answer is: Calculation of correlation coefficient
Question 103
Not answered
Mark 0.00 out of 1.00
Flag question
Question text
A doctor of the general practice has registered the following death causes for the previous year: the
first place was taken by cardiovascular diseases (60%), the second - by tumors (18%), then -
traumas (8,3%) etc. What diagrams will provide the most substantial information about the registered
ocurrences?
Select one:
a. Cartogram
b. Column diagram
c. Pie diagram
d. Line diagram
e. Circle diagram
Feedback
The correct answer is: Pie diagram
Question 104
Not answered
Mark 0.00 out of 1.00
Flag question
Question text
In a forest summer camp children have variable procedures to harden their organisms. What
procedure has the most hardening power?
Select one:
a. Bath with hydromassage
b. Morning exercises on the fresh air
c. Walking on the fresh air
d. Hygienic shower
e. Contrast shower
Feedback
The correct answer is: Contrast shower
Question 105
Not answered
Mark 0.00 out of 1.00
Flag question
Question text
A 42 year old metalworker has been working at the turning machine for production of heavy large-
size parts for 5 years. His work requires using of hand and pedal levers that involves considerable
physical force. What means for osteoarthrosis prevention should be recommended?
Select one:
a. To go in for weightlifting
b. To administer protein-and-vitamin diet
c. To administer protein-and-carbohydrate diet
d. To improve health at the Black sea coast
e. To limit physical work
Feedback
The correct answer is: To limit physical work
Question 106
Not answered
Mark 0.00 out of 1.00
Flag question
Question text
On physiologic-sanitary examination of railway department work it was revealed that loaders work is
of III degree of difficulty. They unload vagons with sand, manually break coagulated mass by shovel
and shift it. What criteria was used to evaluate work of loaders?
Select one:
a. Maximun load weigh which is shifted
b. Time of passive observation, % to the shift duration
c. Intellectual efforts
d. Value of static loading for the shift
e. Time of active activities, % to the shift duration
Feedback
The correct answer is: Maximun load weigh which is shifted
Question 107
Not answered
Mark 0.00 out of 1.00
Flag question
Question text
In an urban settlement situated on the riverbank, an outbreak of hepatitis A was registered. The
disease might have water origin. This assumption can be confirmed by growth of the following
values of water quality:
Select one:
a. Escherichia coli index
b. Index of fecal coli-forms
c. Oxidability
d. Number of coli-phages
e. Presence of benign leptospirosis pathogen
Feedback
The correct answer is: Number of coli-phages
Question 108
Not answered
Mark 0.00 out of 1.00
Flag question
Question text
An engineer-chemist at the age of 47 often fells ill with an occupational skin disease. Who makes a
decision to transfer him to other job accepts?
Select one:
a. A head physician
b. The attending physician
c. MSEC
d. DCC
e. The chief of shop
Feedback
The correct answer is: DCC
Question 109
Not answered
Mark 0.00 out of 1.00
Flag question
Question text
Educational rooms are illuminated with various lighting fittings. What type of lighting fittings is the
most appropriate in respect of hygienic norms?
Select one:
a. Combined light fittings
b. Indirect light fittings
c. Direct light fittings
d. Semi-reflected light fittings
e. Ambient light fittings
Feedback
The correct answer is: Indirect light fittings
Question 110
Not answered
Mark 0.00 out of 1.00
Flag question
Question text
A therapeutist needs to analyze adult health in the area of service. Which groups of indicators will be
included into this analysis?
Select one:
a. Sickness rates, disability, death rates
b. Birth rates, sickness rates, disability
c. Demographic, sickness rates, physical development
d. Sickness rates, death rates, physical development
e. Demographic, sickness rates, disability
Feedback
The correct answer is: Demographic, sickness rates, disability
Question 111
Not answered
Mark 0.00 out of 1.00
Flag question
Question text
Thyreotoxicosis patient is in the two-place hospital ward of therapeutic department. The area of the
ward is 18 m2, height is 3 m, ventilation rate is 2,5/h. Air temperature is 200С, relative humidity is
45%, air movement velocity is 0,3 m/sec, light coefficient is 1/5, noise level constitutes 30 dB. Make
a hygienic assessment of these conditions.
Select one:
a. Poor lighting
b. All conditions are OK
c. Non-effective ventilation
d. Discomfortable microclimate
e. High level of noise
Feedback
The correct answer is: Discomfortable microclimate
Question 112
Not answered
Mark 0.00 out of 1.00
Flag question
Question text
Hygienic expertise of a sample taken from the batch of grain revealed that 2% of grains were
infected with microscopic Fusarium fungi. On the ground of laboratory analyses this batch of grain
should be:
Select one:
a. Used for forage production
b. Sold without restrictions
c. Used for ethanol production
d. Tested for toxicity
e. Destroyed
Feedback
The correct answer is: Sold without restrictions
Question 113
Not answered
Mark 0.00 out of 1.00
Flag question
Question text
A 9 y.o. girl has an average height and harmonic growth development. She was ill with acute
respiratory infection for five times. Define the group of her health.
Select one:
a. 5th group
b. 2nd group
c. 4th group
d. 3rd group
e. 1st group
Feedback
The correct answer is: 2nd group
Question 114
Not answered
Mark 0.00 out of 1.00
Flag question
Question text
Indicate the registration medical document for the patient, who 21.02. was addressed to the doctor
with diagnosis ARVD for the first time in this year:
Select one:
a. The necessary registration form is not indicated
b. The statistical coupon for registration of final diagnosis is not necessary
c. It is necessary to fill in the emergency notice on a case of a contagion
d. The statistical coupon is to be filled in, but a sign (+) is not necessary to be put in
e. The statistical coupon is to be filled in and it is necessary to deliver on a sign (+)
Feedback
The correct answer is: The statistical coupon is to be filled in and it is necessary to deliver on a sign
(+)
Question 115
Not answered
Mark 0.00 out of 1.00
Flag question
Question text
A teacher of a secondary school was diagnosed with pulmonary tuberculosis. What is the maximum
duration of his medical certificate?
Select one:
a. A month
b. Two months
c. Four months
d. Ten months
e. Five months
Feedback
The correct answer is: Ten months
Question 116
Not answered
Mark 0.00 out of 1.00
Flag question
Question text
Estimation of community health level involved analysis of a report on diseases registered among the
population of district under charge (reporting form 12). What index is calculated on the grounds of
this report?
Select one:
a. Common morbidity rate
b. Index of morbidity with temporary disability
c. Index of pathological affection
d. Index of basic non-epidemic morbidity
e. Index of hospitalized morbidity
Feedback
The correct answer is: Common morbidity rate
Question 117
Not answered
Mark 0.00 out of 1.00
Flag question
Question text
A 46-year-old patient was issued a 10-day sick list because of exacerbation of chronic cholecystitis.
The patient's general condition got better, but the clinical manifestations of the disease are still
present. What authority is entitled to extend the sick list?
Select one:
a. Family doctor
b. Deputy head doctor for medical-labour expertise
c. Medical Expert Commission
d. Head doctor
e. Deputy head doctor for terapeutic management
Feedback
The correct answer is: Medical Expert Commission
Question 118
Not answered
Mark 0.00 out of 1.00
Flag question
Question text
Chief district pediatrician has to carry out analysis of infant mortality rate. What should he take as a
unit of the observation?
Select one:
a. Child death case during first 7 days of life
b. Child death case at the age up to the first month
c. Child death case at the age up to 1 year
d. Child death case on labor
e. Child death case after 28 days of life
Feedback
The correct answer is: Child death case at the age up to 1 year
Question 119
Not answered
Mark 0.00 out of 1.00
Flag question
Question text
The district pediatrician is charged with the analysis of infant mortality. What is taken for the unit of
observation in infant mortality investigation?
Select one:
a. A baby dead at the age over 28 days
b. A baby dead at the age up to 6 days
c. A baby dead at the age up to 12 months
d. A baby dead at the age up to 1 months
e. A baby dead at birth
Feedback
The correct answer is: A baby dead at the age up to 12 months
Question 120
Not answered
Mark 0.00 out of 1.00
Flag question
Question text
An emergency situation at a chemical plant caused acute occupational intoxication. A doctor who
revealed the case of "acute occupational disease (intoxication)" must notify the following authority:
Select one:
a. Sanitary and epidemiological station
b. Ministry of Health of Ukraine
c. Medical unit of the plant
d. Plant administration
e. Trade union committee of the plant
Feedback
The correct answer is: Sanitary and epidemiological station
Question 121
Not answered
Mark 0.00 out of 1.00
Flag question
Question text
What methods of the collecting of the information is preferable for study of housing conditions of
students of medical HIGH SCHOOL for a training period?
Select one:
a. Questioning
b. Statistical
c. Selecting of materials
d. A method of the directed selection
e. Interviewing
Feedback
The correct answer is: Questioning
Question 122
Not answered
Mark 0.00 out of 1.00
Flag question
Question text
The average body lenth of newborn boys is 50,9 cm at a sigma 1,66; and average mass - 3432 at a
sigma 5,00. What criterion is necessary in order to compare degree of variability of these signs?
Select one:
a. Sigma
b. Coefficient of association
c. Amplitude
d. Coefficient of variation
e. Limit
Feedback
The correct answer is: Coefficient of variation
Question 123
Not answered
Mark 0.00 out of 1.00
Flag question
Question text
Prevalence of a disease in region N amounted 1156 occurences per 1000 of inhabitants. What of the
mentioned indices characterizes the disease prevalence?
Select one:
a. Intensive
b. Ratio
c. Standardized
d. Visual index
e. Extensive
Feedback
The correct answer is: Intensive
Question 124
Not answered
Mark 0.00 out of 1.00
Flag question
Question text
While asessing the health status of graduates of a secondary school, the doctor found one of them
to have grade 3 tonsillar hypertrophy, chronic rhinitis and vegetative-vascular dystonia. The
organism functionality is reduced. This student belongs to the following health group:
Select one:
a. IV
b. I
c. V
d. III
e. II
Feedback
The correct answer is: III
Question 125
Not answered
Mark 0.00 out of 1.00
Flag question
Question text
The parameter of infantile mortality for the last year was - 16,3, in present year - 15,7. Name a kind
of the diagram that can be used for a graphic representation of it:
Select one:
a. Radial
b. Sector
c. Linear
d. Intrastylar
e. Stylar
Feedback
The correct answer is: Stylar
Question 126
Not answered
Mark 0.00 out of 1.00
Flag question
Question text
In order to study impact of microclimate upon the human organism it is necessary to make
systematic observation of air temperature over 3 days. Choose a device that will allow to make the
most precise temperature records:
Select one:
a. August's psychrometer
b. Alcohol thermometer
c. Thermograph
d. Assmann psychrometer
e. Mercury thermometer
Feedback
The correct answer is: Thermograph
Question 127
Not answered
Mark 0.00 out of 1.00
Flag question
Question text
A patient with high temperature came to a first-aid post in the evening. The fact of temporary
disability was established. Indicate the order of examination in this case:
Select one:
a. The sick list for 3 days should be issued
b. The sick list for up to 3 days should be issued
c. Any document shouldn't be issued
d. The sick list for 1 day should be issued
e. The night duty doctor should issue a medical certificate, which will be subsequently used for
issuing a sick list from the date of the previous day
Feedback
The correct answer is: The night duty doctor should issue a medical certificate, which will be
subsequently used for issuing a sick list from the date of the previous day
Question 128
Not answered
Mark 0.00 out of 1.00
Flag question
Question text
A general practitioner visited a 2-year-old child and diagnosed him with measles. The child attends a
nursery, has a 5-year-old sister. What document must be filled in for the effective antiepidemic
measures in the given health locality?
Select one:
a. Infant's record (report form № 112/o)
b. House call record (form № 031/o)
c. Carer's leave certificate
d. Sick leave
e. Emergency notification on infectious disease (form № 058/o)
Feedback
The correct answer is: Emergency notification on infectious disease (form № 058/o)
Question 129
Not answered
Mark 0.00 out of 1.00
Flag question
Question text
Clinic of a research instutute for occupational diseases examined a worker who works at a
concentration plant and diagnosed him with chronic dust bronchitis. The case is investigated by a
commission including the representatives of: the plant, clinic, territorial SES, department of Social
Insurance Fund, trade union. According to the "regulation on investigation of…", the commission
should be headed by the representative of the following authority:
Select one:
a. Plant
b. Trade union
c. Social Insurance Fund
d. Clinic
e. Territorial SES
Feedback
The correct answer is: Territorial SES
Question 130
Not answered
Mark 0.00 out of 1.00
Flag question
Question text
A 5 tons milk batch was sampled. The lab analysis revealed: fat content 2%, specific density - 1,04
g/cm3, acidity - 210Т, reductase probe - weak-positive. What way is the product to be used in?
Select one:
a. Discard for animal feeding
b. Sell but inform customers about milk quality
c. Sell without limitations
d. Do the product away
e. Technical utilization
Feedback
The correct answer is: Sell but inform customers about milk quality
Question 131
Not answered
Mark 0.00 out of 1.00
Flag question
Question text
During a regular medical examination at a metallurgical plant 20% of workers were found overweight
(body weight was 5-14% higher than normal), and had early signs of obesity (grade I-II) with
Quetelet index from 26 to 30. What products share must be reduced in the diet of this group of
people in the first place in order to normalize their body weight?
Select one:
a. Bakery products
b. Milk and dairy products
c. Vegetables
d. Meat and fish products
e. Fruit
Feedback
The correct answer is: Bakery products
Question 132
Not answered
Mark 0.00 out of 1.00
Flag question
Question text
At a machine-building plant the casts are cleaned by means of abrasion machines that are a source
of local vibration. What are the most efficient preventive measures for preventing harmful effect of
vibration on workers' organisms?
Select one:
a. Preliminary and periodical medical examinations
b. Warm hand baths
c. Use of gloves that reduce vibration
d. Giving sanitary instructions to the workers
e. Hand massaging
Feedback
The correct answer is: Use of gloves that reduce vibration
Question 133
Not answered
Mark 0.00 out of 1.00
Flag question
Question text
A maternity hospital registered 616 live births, 1 stillbirth, 1 death on the 5th day of life over a 1 year
period. What index allows the most precise estimation of this situation?
Select one:
a. Natality
b. Crude mortality rate
c. Natural increase
d. Neonatal mortality
e. Perinatal mortality
Feedback
The correct answer is: Perinatal mortality
Question 134
Not answered
Mark 0.00 out of 1.00
Flag question
Question text
Production areas of a greenhouse complex have the following microclimate parameters: air
temperature - 42oC, humidity - 98%, air velocity - 0,05 mps, temperature of enclosing surfaces -
15oC. Characterize the microclimate of production areas:
Select one:
a. Uncomfortable
b. Satisfactory
c. Cooling
d. Overheated
e. Comfortable
Feedback
The correct answer is: Overheated
Question 135
Not answered
Mark 0.00 out of 1.00
Flag question
Question text
25 children at the age of 2-3 years who don't attend any child welfare institutions should be observed
by a district pediatrician within the current year. How many initial visits of this group of children
should be planned?
Select one:
a. 200
b. 20
c. 40
d. 50
e. 100
Feedback
The correct answer is: 50
Question 136
Not answered
Mark 0.00 out of 1.00
Flag question
Question text
Carpathian region is characterized by permanently high (over 80%) air humidity. In the cold season
the population of this region feels very cold at moderately low temperatures. This is due to an
increase in the heat transfer by:
Select one:
a. Radiation
b. Conduction
c. Emission
d. Convection
e. Evaporation
Feedback
The correct answer is: Convection
Question 137
Not answered
Mark 0.00 out of 1.00
Flag question
Question text
The institutions which take part in medical examinations can be prevention and treatment facilities,
medical board of Ministry of Defense, medical board of Ministry of Home Affairs, medico-social
expert commissions, forensic medical boards etc. What institutions are responsible for temporary
disability examination?
Select one:
a. Prevention and treatment facilities
b. Medical boards of the Ministry of Home Affairs
c. Medico-social expert commissions
d. Sanitary-and-prophylactic institutions
e. Medical boards of the Ministry of Defense
Feedback
The correct answer is: Prevention and treatment facilities
Question 138
Not answered
Mark 0.00 out of 1.00
Flag question
Question text
Basing upon the data of laboratory assessment of sanitary state of soil in a certain territory, the soil
was found to be low-contaminated according to the sanitary indicative value; contaminated
according to the coli titer; low-contaminated according to the anaerobe titer (Cl. Perfringens). This is
indicative of:
Select one:
a. Insufficient intensity of soil humification
b. Insufficient insolation and aeration of soil
c. Old fecal contamination
d. Constant entry of organic protein contaminations
e. Fresh fecal contamination
Feedback
The correct answer is: Fresh fecal contamination
Question 139
Not answered
Mark 0.00 out of 1.00
Flag question
Question text
Workers of a laboratory producing measuring devices (manometers, thermometers etc) complain
about a mettalic taste in mouth, stomatitis, dyspepsia, sleep disturbance, unsteady walk, abrupt
decrease in cardiac activity. These presentations must have been caused by the intoxication with the
following substance:
Select one:
a. Tetraethyl lead
b. Lead
c. Manganese
d. Toluol
e. Mercury
Feedback
The correct answer is: Mercury
Question 140
Not answered
Mark 0.00 out of 1.00
Flag question
Question text
Study of morbidity with temporary disability among workers of a machine building plant revealed that
average duration of a case was 20 days. What diseases influenced upon the index value?
Select one:
a. Acute
b. Subacute
c. Chronic
d. Preexisting diseases
e. Hard to determine
Feedback
The correct answer is: Chronic
Question 141
Not answered
Mark 0.00 out of 1.00
Flag question
Question text
A sample of milk was taken for testing from a 5 ton milk batch. Lab analysis showed the following: fat
content 2%, specific density- 1,04 g/cm3, acidity 210C, reductase probe – weak positive. What way
the product is to be used in? What would you advise?
Select one:
a. Sell without limitations
b. Annihilate the product
c. Sell but inform customers about milk quality
d. Utilize technically
e. Write the product off for animal feeding
Feedback
The correct answer is: Sell but inform customers about milk quality
Question 142
Not answered
Mark 0.00 out of 1.00
Flag question
Question text
It is determined that 30 of a 100 women with risk factor had preterm labor, and of a 100 women
without risk factor 5 women had preterm labor. What method of statistic data processing should the
doctor use in order to estimate reliability of differences between the compared groups?
Select one:
a. Relative numbers calculation
b. Standardization method
c. Average computing
d. Correlation analysis
e. Student's criterion calculation
Feedback
The correct answer is: Student's criterion calculation
Question 143
Not answered
Mark 0.00 out of 1.00
Flag question
Question text
At the radiological unit of a hospital gamma-devices of radiotherapy of "Agat" type and other closed
sources of ionizing radiation are used for treatment of malignant neoplasms. What measures are to
be taken to protect personnel during working with radioactive sources of such type?
Select one:
a. The increase of distance to the source and individual hygiene compliance
b. Reduction of working time and screening of the source
c. Screening of the source and the use of means of individual protection of respiration organs
d. Systematical cleansing of surfaces from the radioactive contamination and shortening of working
time
e. Capsulation of devices and organization of room ventilation
Feedback
The correct answer is: Reduction of working time and screening of the source
Question 144
Not answered
Mark 0.00 out of 1.00
Flag question
Question text
A worker who undergoes regular medical check-up for duodenal ulcer received a subsidized 24-day
sanatorium voucher from his plant. The term of annual leave of a worker is 24 calender days, it will
take 4 days more to get to the sanatorium and back home. What is the procedure of obtaining a 4-
day sick-leave?
Select one:
a. Medical Expert Commission issues a 4-day health certificate
b. The doctor in charge issues a 4-day sick list
c. Medical Expert Commission issues a 4-day sick list
d. Medical Expert Commission issues a 28-day sick list
e. The doctor in charge issues a health certificate and sanatorium patient's file for 28 days
Feedback
The correct answer is: Medical Expert Commission issues a 4-day sick list
Question 145
Not answered
Mark 0.00 out of 1.00
Flag question
Question text
Sanitary-veterinary examination of a cow carcass revealed measle contamination (2-3 measles per
10 cm2). What tactics should be chosen in respect of this meat consumption?
Select one:
a. Meat should be disinfected by freezing
b. The whole carcass should be technically disposed
c. The carcass should be used for the production of canned meat
d. Meat can be sold without any restrictions
e. Meat should be disinfected by boiling
Feedback
The correct answer is: The whole carcass should be technically disposed
Question 146
Not answered
Mark 0.00 out of 1.00
Flag question
Question text
Maximum permissible concentration of carbon dioxide in the air is considered to be a sanitary index
of air purity in a classroom. What concentration of carbon dioxide in the air is accepted as maximum
permissible?
Select one:
a. 0,15%
b. 0,1%
c. 0,2%
d. 0,3%
e. 0,05%
Feedback
The correct answer is: 0,1%
Question 147
Not answered
Mark 0.00 out of 1.00
Flag question
Question text
Define the basic registration document at the profound study of a case rate with temporary lost labor
ability at the industrial enterprise:
Select one:
a. The sick-leave certificate
b. The inpatient medical record
c. A card of the personal account of a case rate
d. "The Report on reasons of a temporary lost labor ability"
e. A ambulatory medical card
Feedback
The correct answer is: A card of the personal account of a case rate
Question 148
Not answered
Mark 0.00 out of 1.00
Flag question
Question text
Estimation of physical development of a child involved dynamometry and estimation of body weight
and length, annual gain in body length, chest circumference, number of permanent teeth, secondary
sexual characters, lung vital capacity. Which of the mentioned indices relates to the physiometric
ones?
Select one:
a. Annual gain in body length
b. Secondary sexual characters
c. Body length and weight, chest circumference
d. Lung vital capacity, dynamometry
e. Number of permanent teeth
Feedback
The correct answer is: Lung vital capacity, dynamometry
Question 149
Not answered
Mark 0.00 out of 1.00
Flag question
Question text
A student analyzes noise level of cold-pressing process. What device should be applied for this
hygienic study?
Select one:
a. Noise and vibration analyzer
b. Pyranometer
c. Sound tester
d. Actinometer
e. Noise analyzer
Feedback
The correct answer is: Noise and vibration analyzer
Question 150
Not answered
Mark 0.00 out of 1.00
Flag question
Question text
In the current year general practitioners of the municipal polyclinic have referred 11 patients with
coronary artery disease to the in-patient hospital. In 3 cases the diagnosis wasn't confirmed. What
managerial decision shoud be made in such case?
Select one:
a. Analysis of diagnostic examination quality
b. Analysis of doctors' skill level
c. Analysis of medical check-up quality
d. Analysis of each case of diagnostic divergence
e. Analysis of material and technical basisof the polyclinic
Feedback
The correct answer is: Analysis of each case of diagnostic divergence
Question 151
Not answered
Mark 0.00 out of 1.00
Flag question
Question text
In order to reduce weed growth on agricultural land, some herbicides have been used for a long
time. In terms of environmental stability these herbicides are rated as stable. Specify the most likely
route of their entry into the human body:
Select one:
a. Soil-animals-humans
b. Soil-microorganisms-humans
c. Soil-protozoa-humans
d. Soil-plants-humans
e. Soil-insects-humans
Feedback
The correct answer is: Soil-plants-humans
Question 152
Not answered
Mark 0.00 out of 1.00
Flag question
Question text
During hygienic examination of a hospital it was established that the area for each bed in a double
ward was: in the infectious department for children - 7 м2, in the infectious department for adults - 8
м2, in the burns department - 9 м2, in the radiological department - 10 м2, in the critical care
department - 13 м2. In which wards the area for each bed doesn't correspond with hygienic
requirements?
Select one:
a. In critical care wards
b. In infectious wards for children
c. In radiological wards
d. In infectious wards for adults
e. In burns wards
Feedback
The correct answer is: In burns wards
Question 153
Not answered
Mark 0.00 out of 1.00
Flag question
Question text
Evaluation results of sanitary and hygiene conditions in a 4-bed ward were as follows: ward area -
30 m2, height - 3,2 m, temperature - 20oC, humidity - 55%, air velocity - 0,1 m/s, window-to-floor
area ratio - 1:5, daylight ratio - 0,6%, concentration of carbon dioxide in the air - 0,1%. Which of the
given indicators does not meet hygienic requirements?
Select one:
a. Air velocity
b. Daylight ratio
c. Concentration of carbon dioxide in the air
d. Window-to-floor area ratio
e. Ward area
Feedback
The correct answer is: Daylight ratio
Question 154
Not answered
Mark 0.00 out of 1.00
Flag question
Question text
The objective of a statistical study was to find out the extent of seeking medical care by the
population. For this purpose 300 residents of the area were interviewed. Information was collected
by means of a special questionnaire. What method of collecting information was used by
researchers?
Select one:
a. Doing extracts
b. Immediate registration
c. Anamnestic
d. Immediate examination
Feedback
The correct answer is: Anamnestic
Question 155
Not answered
Mark 0.00 out of 1.00
Flag question
Question text
At's planned to construct multifield a new hospital in one of the cental city districts. What building
type is the most appropriate in this case?
Select one:
a. Decentralized
b. Mixed
c. Blocked
d. Centralized and blocked
e. Centralized
Feedback
The correct answer is: Centralized and blocked
Question 156
Not answered
Mark 0.00 out of 1.00
Flag question
Question text
It is planned to build a multi-disciplinary hospital with 500 beds in a town. Specify the location of a
polyclinic within the medical centre:
Select one:
a. At the main entrance
b. In the service zone
c. It is not allowed to place the polyclinic within the centre territory
d. In the centre of the territory near medical buildings
e. In the garden and park area
Feedback
The correct answer is: At the main entrance
Question 157
Not answered
Mark 0.00 out of 1.00
Flag question
Question text
A patient who had eaten mushrooms in the morning was delivered to the infectious diseases hospital
at night. The disease development was rapid. The patient presented with stomach pain, frequent
diarrhea, intractable vomiting, burning thirst, headache and dizziness. He died on the third day. What
mushrooms are most likely to have caused mycetismus?
Select one:
a. Sulfur-tufts
b. Deadly amanita
c. Russules
d. Morels
e. Fly agarics
Feedback
The correct answer is: Deadly amanita
Question 158
Not answered
Mark 0.00 out of 1.00
Flag question
Question text
A patient is on the sick leave for 4 months continuously from the date of injury. The treatment is
going to last for 1-2 months more. Who has the right to extend the duration of medical certificate for
this patient?
Select one:
a. Medical superintendent
b. Medical advisory commission after inpatient treatment
c. Medico-social expert commission
d. Medical advisory commission after medico-social expert commission examination
e. District doctor by agreement with a department chief
Feedback
The correct answer is: Medical advisory commission after medico-social expert commission
examination
Question 159
Not answered
Mark 0.00 out of 1.00
Flag question
Question text
Examination of a 43-year-old man objectively revealed pallor of skin and mucous membranes, loss
of tongue papillae, transverse striation of fingernails, cracks in the mouth corners, tachycardia. Blood
test results: Hb- 90 g/l, anisocytosis, poikilocytosis. The most likely causative agent of this state is
inadequate intake of:
Select one:
a. Zinc
b. Iron
c. Selene
d. Magnesium
e. Copper
Feedback
The correct answer is: Iron
Question 160
Not answered
Mark 0.00 out of 1.00
Flag question
Question text
A city somatic hospital with 300 beds consists of the main building which houses the therapeutic and
surgical departments. Several separate buildings house the maternity, pediatric and radiologic
departments that are connected to the main building by underground walkways and above-ground
covered skybridges. Specify the building system of the hospital:
Select one:
a. Decentralized
b. Combined
c. Free
d. Central-unit
e. Centralized
Feedback
The correct answer is: Central-unit
Question 161
Not answered
Mark 0.00 out of 1.00
Flag question
Question text
The correlation between the service record and eosinophil concentration in blood was studied in
workers at dyeing shops of textile factories. What index will be the most informative for the analysis
of this data?
Select one:
a. Sign index
b. Fitting criterion
c. Student's criterion
d. Correlation factor
e. Standardized index
Feedback
The correct answer is: Correlation factor
Question 162
Not answered
Mark 0.00 out of 1.00
Flag question
Question text
Indices that characterize population health include demographic indices. What environment is used
for calculation of these indices?
Select one:
a. Number of hospitalized people
b. Employment number
c. Number of population being liable to preventive examination
d. Population number
e. Number of patients
Feedback
The correct answer is: Population number
Question 163
Not answered
Mark 0.00 out of 1.00
Flag question
Question text
A number of viable fetuses per 1000 women at the age between 15 and 44 is determined by:
Select one:
a. Reproductive level
b. Obstetric rate
c. Perinatal rate
d. Birth rate
e. Genital index
Feedback
The correct answer is: Genital index
Question 164
Not answered
Mark 0.00 out of 1.00
Flag question
Question text
It is required to analyze the level of daylight illumination in a ward of therapeutics unit. What device
should be applied to estimate the level of daylight illumination?
Select one:
a. Psychrometer
b. Katathermometer
c. Actinometer
d. Anemometer
e. Illuminometer
Feedback
The correct answer is: Illuminometer
Question 165
Not answered
Mark 0.00 out of 1.00
Flag question
Question text
Clinical and statistical study was devoted to the effect of a new pharmacological medication upon the
patients with coronary heart disease. What parametric criterion (coefficient) can be used for
estimation of results validity?
Select one:
a. Sign criterion
b. Student's coefficient (t)
c. Kolmogorov-Smirnov's criterion
d. Conformity coefficient
e. Wilcoxon's t-criterion
Feedback
The correct answer is: Student's coefficient (t)

More Related Content

What's hot

Krok 1 - 2010 Question Paper (Stomatology)
Krok 1 - 2010 Question Paper (Stomatology)Krok 1 - 2010 Question Paper (Stomatology)
Krok 1 - 2010 Question Paper (Stomatology)Eneutron
 
Krok 1 - 2015 Question Paper (General medicine)
Krok 1 - 2015 Question Paper (General medicine)Krok 1 - 2015 Question Paper (General medicine)
Krok 1 - 2015 Question Paper (General medicine)Eneutron
 
The respiratory system
The respiratory systemThe respiratory system
The respiratory systemkevperrino
 
Integration of nervous system functions
Integration of nervous system functionsIntegration of nervous system functions
Integration of nervous system functionskevperrino
 
Respiratory mcq rdmc_2018
Respiratory mcq rdmc_2018Respiratory mcq rdmc_2018
Respiratory mcq rdmc_2018Parthiv Mehta
 
Pediatric Arab Board MCQ Review - Emergency Medicine
Pediatric Arab Board MCQ Review - Emergency Medicine Pediatric Arab Board MCQ Review - Emergency Medicine
Pediatric Arab Board MCQ Review - Emergency Medicine Fatima Farid
 
Krok 2 - 2014 (Hygiene)
Krok 2 - 2014 (Hygiene)Krok 2 - 2014 (Hygiene)
Krok 2 - 2014 (Hygiene)Eneutron
 
Krok 1 - 2011 Question Paper (General medicine)
Krok 1 - 2011 Question Paper (General medicine)Krok 1 - 2011 Question Paper (General medicine)
Krok 1 - 2011 Question Paper (General medicine)Eneutron
 
Questions 1700 - Medical MCQ without Answer
Questions 1700 - Medical MCQ without AnswerQuestions 1700 - Medical MCQ without Answer
Questions 1700 - Medical MCQ without AnswerMBBS Help
 
Pediatric 188 krok 2
Pediatric 188 krok 2Pediatric 188 krok 2
Pediatric 188 krok 2Raj Twix
 
Ch 23 quiz_show_game
Ch 23 quiz_show_gameCh 23 quiz_show_game
Ch 23 quiz_show_gamekevperrino
 
MCQs respiratory system
MCQs respiratory systemMCQs respiratory system
MCQs respiratory systemDOCTOR WHO
 
Krok 2 Medicine - 2017 Question Paper
Krok 2 Medicine - 2017 Question PaperKrok 2 Medicine - 2017 Question Paper
Krok 2 Medicine - 2017 Question PaperEneutron
 
11 quiz show
11 quiz show11 quiz show
11 quiz showTheSlaps
 
14 quiz show
14 quiz show14 quiz show
14 quiz showTheSlaps
 

What's hot (20)

24 quiz-show
24 quiz-show24 quiz-show
24 quiz-show
 
Krok 1 - 2010 Question Paper (Stomatology)
Krok 1 - 2010 Question Paper (Stomatology)Krok 1 - 2010 Question Paper (Stomatology)
Krok 1 - 2010 Question Paper (Stomatology)
 
Krok 1 - 2015 Question Paper (General medicine)
Krok 1 - 2015 Question Paper (General medicine)Krok 1 - 2015 Question Paper (General medicine)
Krok 1 - 2015 Question Paper (General medicine)
 
The respiratory system
The respiratory systemThe respiratory system
The respiratory system
 
Integration of nervous system functions
Integration of nervous system functionsIntegration of nervous system functions
Integration of nervous system functions
 
Respiratory mcq rdmc_2018
Respiratory mcq rdmc_2018Respiratory mcq rdmc_2018
Respiratory mcq rdmc_2018
 
Pediatric Arab Board MCQ Review - Emergency Medicine
Pediatric Arab Board MCQ Review - Emergency Medicine Pediatric Arab Board MCQ Review - Emergency Medicine
Pediatric Arab Board MCQ Review - Emergency Medicine
 
Ganyang MCQ Respiratory
Ganyang MCQ RespiratoryGanyang MCQ Respiratory
Ganyang MCQ Respiratory
 
Krok 2 - 2014 (Hygiene)
Krok 2 - 2014 (Hygiene)Krok 2 - 2014 (Hygiene)
Krok 2 - 2014 (Hygiene)
 
Krok 1 - 2011 Question Paper (General medicine)
Krok 1 - 2011 Question Paper (General medicine)Krok 1 - 2011 Question Paper (General medicine)
Krok 1 - 2011 Question Paper (General medicine)
 
Questions 1700 - Medical MCQ without Answer
Questions 1700 - Medical MCQ without AnswerQuestions 1700 - Medical MCQ without Answer
Questions 1700 - Medical MCQ without Answer
 
Bronchiectasis
BronchiectasisBronchiectasis
Bronchiectasis
 
Pediatric 188 krok 2
Pediatric 188 krok 2Pediatric 188 krok 2
Pediatric 188 krok 2
 
Mcqs cvs 2
Mcqs cvs 2Mcqs cvs 2
Mcqs cvs 2
 
Ch 23 quiz_show_game
Ch 23 quiz_show_gameCh 23 quiz_show_game
Ch 23 quiz_show_game
 
MCQs respiratory system
MCQs respiratory systemMCQs respiratory system
MCQs respiratory system
 
Krok 2 Medicine - 2017 Question Paper
Krok 2 Medicine - 2017 Question PaperKrok 2 Medicine - 2017 Question Paper
Krok 2 Medicine - 2017 Question Paper
 
11 quiz show
11 quiz show11 quiz show
11 quiz show
 
14 quiz show
14 quiz show14 quiz show
14 quiz show
 
Hepatopulmonary syndrome
Hepatopulmonary syndromeHepatopulmonary syndrome
Hepatopulmonary syndrome
 

Viewers also liked

Hygien 2013 krok 2
Hygien 2013 krok 2Hygien 2013 krok 2
Hygien 2013 krok 2Raj Twix
 
Hygienic addi 134 krok 2
Hygienic addi 134 krok 2Hygienic addi 134 krok 2
Hygienic addi 134 krok 2Raj Twix
 
Gynac 170mcq krok 2
Gynac 170mcq krok 2Gynac 170mcq krok 2
Gynac 170mcq krok 2Raj Twix
 
Three dimensions of information systems
Three dimensions of information systemsThree dimensions of information systems
Three dimensions of information systemsSuleyman Ally
 
Book 2007 krok 2
Book 2007 krok 2Book 2007 krok 2
Book 2007 krok 2Raj Twix
 
Endo Pat2000 Sales Presentation
Endo Pat2000 Sales PresentationEndo Pat2000 Sales Presentation
Endo Pat2000 Sales Presentationrllalpha
 
Joomla e seo reifison
Joomla e seo   reifisonJoomla e seo   reifison
Joomla e seo reifisonreifison
 
Simple english grammar
Simple english grammarSimple english grammar
Simple english grammarrestuputraku5
 
Enhanced Computer Vision with Microsoft Kinect Sensor: A Review
Enhanced Computer Vision with Microsoft Kinect Sensor: A ReviewEnhanced Computer Vision with Microsoft Kinect Sensor: A Review
Enhanced Computer Vision with Microsoft Kinect Sensor: A ReviewAbu Saleh Musa
 
Analyze what happens to the market for pizza, if the price of tomato rises an...
Analyze what happens to the market for pizza, if the price of tomato rises an...Analyze what happens to the market for pizza, if the price of tomato rises an...
Analyze what happens to the market for pizza, if the price of tomato rises an...Tanzila Islam
 
next to of course god america i by EE Cummings
next to of course god america i by EE Cummingsnext to of course god america i by EE Cummings
next to of course god america i by EE Cummingswww.MrSedani.co.uk
 
Using the SDACK Architecture to Build a Big Data Product
Using the SDACK Architecture to Build a Big Data ProductUsing the SDACK Architecture to Build a Big Data Product
Using the SDACK Architecture to Build a Big Data ProductEvans Ye
 
Blue star package units
Blue star package unitsBlue star package units
Blue star package unitsKumar Ramaiah
 
Project Management ORION Systems
Project Management ORION SystemsProject Management ORION Systems
Project Management ORION Systemsreeza fazily
 
Business etiquette case study
Business etiquette case studyBusiness etiquette case study
Business etiquette case studysahoosc
 

Viewers also liked (20)

Hygien 2013 krok 2
Hygien 2013 krok 2Hygien 2013 krok 2
Hygien 2013 krok 2
 
Hygienic addi 134 krok 2
Hygienic addi 134 krok 2Hygienic addi 134 krok 2
Hygienic addi 134 krok 2
 
Gynac 170mcq krok 2
Gynac 170mcq krok 2Gynac 170mcq krok 2
Gynac 170mcq krok 2
 
Three dimensions of information systems
Three dimensions of information systemsThree dimensions of information systems
Three dimensions of information systems
 
Book 2007 krok 2
Book 2007 krok 2Book 2007 krok 2
Book 2007 krok 2
 
Changes in me
Changes in meChanges in me
Changes in me
 
Endo Pat2000 Sales Presentation
Endo Pat2000 Sales PresentationEndo Pat2000 Sales Presentation
Endo Pat2000 Sales Presentation
 
Joomla e seo reifison
Joomla e seo   reifisonJoomla e seo   reifison
Joomla e seo reifison
 
Simple english grammar
Simple english grammarSimple english grammar
Simple english grammar
 
Enhanced Computer Vision with Microsoft Kinect Sensor: A Review
Enhanced Computer Vision with Microsoft Kinect Sensor: A ReviewEnhanced Computer Vision with Microsoft Kinect Sensor: A Review
Enhanced Computer Vision with Microsoft Kinect Sensor: A Review
 
Analyze what happens to the market for pizza, if the price of tomato rises an...
Analyze what happens to the market for pizza, if the price of tomato rises an...Analyze what happens to the market for pizza, if the price of tomato rises an...
Analyze what happens to the market for pizza, if the price of tomato rises an...
 
next to of course god america i by EE Cummings
next to of course god america i by EE Cummingsnext to of course god america i by EE Cummings
next to of course god america i by EE Cummings
 
At the Border by Choman Hardi
At the Border by Choman HardiAt the Border by Choman Hardi
At the Border by Choman Hardi
 
Gangguan belajar
Gangguan belajarGangguan belajar
Gangguan belajar
 
Using the SDACK Architecture to Build a Big Data Product
Using the SDACK Architecture to Build a Big Data ProductUsing the SDACK Architecture to Build a Big Data Product
Using the SDACK Architecture to Build a Big Data Product
 
Unit 2 powerpoint
Unit 2 powerpointUnit 2 powerpoint
Unit 2 powerpoint
 
Blue star package units
Blue star package unitsBlue star package units
Blue star package units
 
Makalah keluarga berencana
Makalah keluarga berencanaMakalah keluarga berencana
Makalah keluarga berencana
 
Project Management ORION Systems
Project Management ORION SystemsProject Management ORION Systems
Project Management ORION Systems
 
Business etiquette case study
Business etiquette case studyBusiness etiquette case study
Business etiquette case study
 

Similar to Hygien 165 krok 2

Book 2011 krok 2
Book 2011 krok 2Book 2011 krok 2
Book 2011 krok 2Raj Twix
 
Krok 2 - 2013 (Hygiene)
Krok 2 - 2013 (Hygiene)Krok 2 - 2013 (Hygiene)
Krok 2 - 2013 (Hygiene)Eneutron
 
Book 2009 krok 2
Book 2009 krok 2Book 2009 krok 2
Book 2009 krok 2Raj Twix
 
Pediatric additional krok 2
Pediatric additional krok 2Pediatric additional krok 2
Pediatric additional krok 2Raj Twix
 
Booklet 2015 krok 2
Booklet 2015 krok 2Booklet 2015 krok 2
Booklet 2015 krok 2Raj Twix
 
Book 2008 krok 2
Book 2008 krok 2Book 2008 krok 2
Book 2008 krok 2Raj Twix
 
Module 11 (concepts of health and disease)
Module 11 (concepts of health and disease)Module 11 (concepts of health and disease)
Module 11 (concepts of health and disease)sallamahmed1
 
Therapeutic 176 additional krok 2
Therapeutic 176 additional krok 2Therapeutic 176 additional krok 2
Therapeutic 176 additional krok 2Raj Twix
 
1. Toxic substances can be described by their ability to cause un
1.  Toxic substances can be described by their ability to cause un1.  Toxic substances can be described by their ability to cause un
1. Toxic substances can be described by their ability to cause unsandibabcock
 
AACN PCCN Adult Certification Exam Your Path to Progressive Care Nursing Success
AACN PCCN Adult Certification Exam Your Path to Progressive Care Nursing SuccessAACN PCCN Adult Certification Exam Your Path to Progressive Care Nursing Success
AACN PCCN Adult Certification Exam Your Path to Progressive Care Nursing SuccessAliza Oscar
 
1.Questions 1 and 2 are based on the following informationYou.docx
1.Questions 1 and 2 are based on the following informationYou.docx1.Questions 1 and 2 are based on the following informationYou.docx
1.Questions 1 and 2 are based on the following informationYou.docxchristiandean12115
 
advanced biostat.pdf
advanced biostat.pdfadvanced biostat.pdf
advanced biostat.pdfGadisaFitala
 
advanced biostat.pdf
advanced biostat.pdfadvanced biostat.pdf
advanced biostat.pdfGadisaFitala
 
Results DisplayedFeedback· Question 10 out of 2 points.docx
Results DisplayedFeedback· Question 10 out of 2 points.docxResults DisplayedFeedback· Question 10 out of 2 points.docx
Results DisplayedFeedback· Question 10 out of 2 points.docxzmark3
 
Surgey addi krok 2
Surgey addi krok 2Surgey addi krok 2
Surgey addi krok 2Raj Twix
 
ANTI TOBACCO DAY essay.docx
ANTI TOBACCO DAY essay.docxANTI TOBACCO DAY essay.docx
ANTI TOBACCO DAY essay.docxanjalatchi
 
General Education Assessment MATH 1314InstructionsDo not inc
General Education Assessment MATH 1314InstructionsDo not incGeneral Education Assessment MATH 1314InstructionsDo not inc
General Education Assessment MATH 1314InstructionsDo not incMatthewTennant613
 
Questionnaire Designing
Questionnaire DesigningQuestionnaire Designing
Questionnaire DesigningJugal Kishore
 

Similar to Hygien 165 krok 2 (20)

Book 2011 krok 2
Book 2011 krok 2Book 2011 krok 2
Book 2011 krok 2
 
Krok 2 - 2013 (Hygiene)
Krok 2 - 2013 (Hygiene)Krok 2 - 2013 (Hygiene)
Krok 2 - 2013 (Hygiene)
 
Book 2009 krok 2
Book 2009 krok 2Book 2009 krok 2
Book 2009 krok 2
 
Pediatric additional krok 2
Pediatric additional krok 2Pediatric additional krok 2
Pediatric additional krok 2
 
Booklet 2015 krok 2
Booklet 2015 krok 2Booklet 2015 krok 2
Booklet 2015 krok 2
 
Book 2008 krok 2
Book 2008 krok 2Book 2008 krok 2
Book 2008 krok 2
 
Module 11 (concepts of health and disease)
Module 11 (concepts of health and disease)Module 11 (concepts of health and disease)
Module 11 (concepts of health and disease)
 
Therapeutic 176 additional krok 2
Therapeutic 176 additional krok 2Therapeutic 176 additional krok 2
Therapeutic 176 additional krok 2
 
1. Toxic substances can be described by their ability to cause un
1.  Toxic substances can be described by their ability to cause un1.  Toxic substances can be described by their ability to cause un
1. Toxic substances can be described by their ability to cause un
 
AACN PCCN Adult Certification Exam Your Path to Progressive Care Nursing Success
AACN PCCN Adult Certification Exam Your Path to Progressive Care Nursing SuccessAACN PCCN Adult Certification Exam Your Path to Progressive Care Nursing Success
AACN PCCN Adult Certification Exam Your Path to Progressive Care Nursing Success
 
1.Questions 1 and 2 are based on the following informationYou.docx
1.Questions 1 and 2 are based on the following informationYou.docx1.Questions 1 and 2 are based on the following informationYou.docx
1.Questions 1 and 2 are based on the following informationYou.docx
 
Fundamental of nursing review questions
Fundamental of nursing review questionsFundamental of nursing review questions
Fundamental of nursing review questions
 
advanced biostat.pdf
advanced biostat.pdfadvanced biostat.pdf
advanced biostat.pdf
 
advanced biostat.pdf
advanced biostat.pdfadvanced biostat.pdf
advanced biostat.pdf
 
Results DisplayedFeedback· Question 10 out of 2 points.docx
Results DisplayedFeedback· Question 10 out of 2 points.docxResults DisplayedFeedback· Question 10 out of 2 points.docx
Results DisplayedFeedback· Question 10 out of 2 points.docx
 
Surgey addi krok 2
Surgey addi krok 2Surgey addi krok 2
Surgey addi krok 2
 
ANTI TOBACCO DAY essay.docx
ANTI TOBACCO DAY essay.docxANTI TOBACCO DAY essay.docx
ANTI TOBACCO DAY essay.docx
 
Public health
Public healthPublic health
Public health
 
General Education Assessment MATH 1314InstructionsDo not inc
General Education Assessment MATH 1314InstructionsDo not incGeneral Education Assessment MATH 1314InstructionsDo not inc
General Education Assessment MATH 1314InstructionsDo not inc
 
Questionnaire Designing
Questionnaire DesigningQuestionnaire Designing
Questionnaire Designing
 

More from Raj Twix

Therapy 2013 krok 2
Therapy 2013 krok 2Therapy 2013 krok 2
Therapy 2013 krok 2Raj Twix
 
Therapeutic 271 krok 2
Therapeutic 271 krok 2Therapeutic 271 krok 2
Therapeutic 271 krok 2Raj Twix
 
Surgery 209 krok 2
Surgery 209 krok 2Surgery 209 krok 2
Surgery 209 krok 2Raj Twix
 
Gync additional krok 2
Gync additional krok 2Gync additional krok 2
Gync additional krok 2Raj Twix
 
Booklet 2013 krok 2
Booklet 2013 krok 2Booklet 2013 krok 2
Booklet 2013 krok 2Raj Twix
 
Book 2006 krok-2
Book 2006 krok-2Book 2006 krok-2
Book 2006 krok-2Raj Twix
 
Occupational book internal medicine 5th year BSMU
Occupational book internal medicine 5th year BSMUOccupational book internal medicine 5th year BSMU
Occupational book internal medicine 5th year BSMURaj Twix
 
Pulmonology book internal medicine 5th year BSMU
Pulmonology book internal medicine 5th year BSMUPulmonology book internal medicine 5th year BSMU
Pulmonology book internal medicine 5th year BSMURaj Twix
 
Cardiology book internal medicine 5th year
Cardiology book internal medicine 5th yearCardiology book internal medicine 5th year
Cardiology book internal medicine 5th yearRaj Twix
 
Hematology-urgent condition book internal medicine 5th year BSMU
Hematology-urgent condition book internal medicine 5th year BSMUHematology-urgent condition book internal medicine 5th year BSMU
Hematology-urgent condition book internal medicine 5th year BSMURaj Twix
 
Microbiology Virology book MBBS
Microbiology Virology book MBBSMicrobiology Virology book MBBS
Microbiology Virology book MBBSRaj Twix
 
Pathomorhology
PathomorhologyPathomorhology
PathomorhologyRaj Twix
 

More from Raj Twix (12)

Therapy 2013 krok 2
Therapy 2013 krok 2Therapy 2013 krok 2
Therapy 2013 krok 2
 
Therapeutic 271 krok 2
Therapeutic 271 krok 2Therapeutic 271 krok 2
Therapeutic 271 krok 2
 
Surgery 209 krok 2
Surgery 209 krok 2Surgery 209 krok 2
Surgery 209 krok 2
 
Gync additional krok 2
Gync additional krok 2Gync additional krok 2
Gync additional krok 2
 
Booklet 2013 krok 2
Booklet 2013 krok 2Booklet 2013 krok 2
Booklet 2013 krok 2
 
Book 2006 krok-2
Book 2006 krok-2Book 2006 krok-2
Book 2006 krok-2
 
Occupational book internal medicine 5th year BSMU
Occupational book internal medicine 5th year BSMUOccupational book internal medicine 5th year BSMU
Occupational book internal medicine 5th year BSMU
 
Pulmonology book internal medicine 5th year BSMU
Pulmonology book internal medicine 5th year BSMUPulmonology book internal medicine 5th year BSMU
Pulmonology book internal medicine 5th year BSMU
 
Cardiology book internal medicine 5th year
Cardiology book internal medicine 5th yearCardiology book internal medicine 5th year
Cardiology book internal medicine 5th year
 
Hematology-urgent condition book internal medicine 5th year BSMU
Hematology-urgent condition book internal medicine 5th year BSMUHematology-urgent condition book internal medicine 5th year BSMU
Hematology-urgent condition book internal medicine 5th year BSMU
 
Microbiology Virology book MBBS
Microbiology Virology book MBBSMicrobiology Virology book MBBS
Microbiology Virology book MBBS
 
Pathomorhology
PathomorhologyPathomorhology
Pathomorhology
 

Recently uploaded

How to Configure Email Server in Odoo 17
How to Configure Email Server in Odoo 17How to Configure Email Server in Odoo 17
How to Configure Email Server in Odoo 17Celine George
 
Enzyme, Pharmaceutical Aids, Miscellaneous Last Part of Chapter no 5th.pdf
Enzyme, Pharmaceutical Aids, Miscellaneous Last Part of Chapter no 5th.pdfEnzyme, Pharmaceutical Aids, Miscellaneous Last Part of Chapter no 5th.pdf
Enzyme, Pharmaceutical Aids, Miscellaneous Last Part of Chapter no 5th.pdfSumit Tiwari
 
Organic Name Reactions for the students and aspirants of Chemistry12th.pptx
Organic Name Reactions  for the students and aspirants of Chemistry12th.pptxOrganic Name Reactions  for the students and aspirants of Chemistry12th.pptx
Organic Name Reactions for the students and aspirants of Chemistry12th.pptxVS Mahajan Coaching Centre
 
Introduction to ArtificiaI Intelligence in Higher Education
Introduction to ArtificiaI Intelligence in Higher EducationIntroduction to ArtificiaI Intelligence in Higher Education
Introduction to ArtificiaI Intelligence in Higher Educationpboyjonauth
 
Call Girls in Dwarka Mor Delhi Contact Us 9654467111
Call Girls in Dwarka Mor Delhi Contact Us 9654467111Call Girls in Dwarka Mor Delhi Contact Us 9654467111
Call Girls in Dwarka Mor Delhi Contact Us 9654467111Sapana Sha
 
Final demo Grade 9 for demo Plan dessert.pptx
Final demo Grade 9 for demo Plan dessert.pptxFinal demo Grade 9 for demo Plan dessert.pptx
Final demo Grade 9 for demo Plan dessert.pptxAvyJaneVismanos
 
Presiding Officer Training module 2024 lok sabha elections
Presiding Officer Training module 2024 lok sabha electionsPresiding Officer Training module 2024 lok sabha elections
Presiding Officer Training module 2024 lok sabha electionsanshu789521
 
Kisan Call Centre - To harness potential of ICT in Agriculture by answer farm...
Kisan Call Centre - To harness potential of ICT in Agriculture by answer farm...Kisan Call Centre - To harness potential of ICT in Agriculture by answer farm...
Kisan Call Centre - To harness potential of ICT in Agriculture by answer farm...Krashi Coaching
 
Employee wellbeing at the workplace.pptx
Employee wellbeing at the workplace.pptxEmployee wellbeing at the workplace.pptx
Employee wellbeing at the workplace.pptxNirmalaLoungPoorunde1
 
Painted Grey Ware.pptx, PGW Culture of India
Painted Grey Ware.pptx, PGW Culture of IndiaPainted Grey Ware.pptx, PGW Culture of India
Painted Grey Ware.pptx, PGW Culture of IndiaVirag Sontakke
 
internship ppt on smartinternz platform as salesforce developer
internship ppt on smartinternz platform as salesforce developerinternship ppt on smartinternz platform as salesforce developer
internship ppt on smartinternz platform as salesforce developerunnathinaik
 
Software Engineering Methodologies (overview)
Software Engineering Methodologies (overview)Software Engineering Methodologies (overview)
Software Engineering Methodologies (overview)eniolaolutunde
 
The Most Excellent Way | 1 Corinthians 13
The Most Excellent Way | 1 Corinthians 13The Most Excellent Way | 1 Corinthians 13
The Most Excellent Way | 1 Corinthians 13Steve Thomason
 
CARE OF CHILD IN INCUBATOR..........pptx
CARE OF CHILD IN INCUBATOR..........pptxCARE OF CHILD IN INCUBATOR..........pptx
CARE OF CHILD IN INCUBATOR..........pptxGaneshChakor2
 
Pharmacognosy Flower 3. Compositae 2023.pdf
Pharmacognosy Flower 3. Compositae 2023.pdfPharmacognosy Flower 3. Compositae 2023.pdf
Pharmacognosy Flower 3. Compositae 2023.pdfMahmoud M. Sallam
 
POINT- BIOCHEMISTRY SEM 2 ENZYMES UNIT 5.pptx
POINT- BIOCHEMISTRY SEM 2 ENZYMES UNIT 5.pptxPOINT- BIOCHEMISTRY SEM 2 ENZYMES UNIT 5.pptx
POINT- BIOCHEMISTRY SEM 2 ENZYMES UNIT 5.pptxSayali Powar
 
Class 11 Legal Studies Ch-1 Concept of State .pdf
Class 11 Legal Studies Ch-1 Concept of State .pdfClass 11 Legal Studies Ch-1 Concept of State .pdf
Class 11 Legal Studies Ch-1 Concept of State .pdfakmcokerachita
 
Science lesson Moon for 4th quarter lesson
Science lesson Moon for 4th quarter lessonScience lesson Moon for 4th quarter lesson
Science lesson Moon for 4th quarter lessonJericReyAuditor
 

Recently uploaded (20)

How to Configure Email Server in Odoo 17
How to Configure Email Server in Odoo 17How to Configure Email Server in Odoo 17
How to Configure Email Server in Odoo 17
 
Enzyme, Pharmaceutical Aids, Miscellaneous Last Part of Chapter no 5th.pdf
Enzyme, Pharmaceutical Aids, Miscellaneous Last Part of Chapter no 5th.pdfEnzyme, Pharmaceutical Aids, Miscellaneous Last Part of Chapter no 5th.pdf
Enzyme, Pharmaceutical Aids, Miscellaneous Last Part of Chapter no 5th.pdf
 
Organic Name Reactions for the students and aspirants of Chemistry12th.pptx
Organic Name Reactions  for the students and aspirants of Chemistry12th.pptxOrganic Name Reactions  for the students and aspirants of Chemistry12th.pptx
Organic Name Reactions for the students and aspirants of Chemistry12th.pptx
 
Model Call Girl in Tilak Nagar Delhi reach out to us at 🔝9953056974🔝
Model Call Girl in Tilak Nagar Delhi reach out to us at 🔝9953056974🔝Model Call Girl in Tilak Nagar Delhi reach out to us at 🔝9953056974🔝
Model Call Girl in Tilak Nagar Delhi reach out to us at 🔝9953056974🔝
 
Introduction to ArtificiaI Intelligence in Higher Education
Introduction to ArtificiaI Intelligence in Higher EducationIntroduction to ArtificiaI Intelligence in Higher Education
Introduction to ArtificiaI Intelligence in Higher Education
 
Call Girls in Dwarka Mor Delhi Contact Us 9654467111
Call Girls in Dwarka Mor Delhi Contact Us 9654467111Call Girls in Dwarka Mor Delhi Contact Us 9654467111
Call Girls in Dwarka Mor Delhi Contact Us 9654467111
 
Final demo Grade 9 for demo Plan dessert.pptx
Final demo Grade 9 for demo Plan dessert.pptxFinal demo Grade 9 for demo Plan dessert.pptx
Final demo Grade 9 for demo Plan dessert.pptx
 
Presiding Officer Training module 2024 lok sabha elections
Presiding Officer Training module 2024 lok sabha electionsPresiding Officer Training module 2024 lok sabha elections
Presiding Officer Training module 2024 lok sabha elections
 
Kisan Call Centre - To harness potential of ICT in Agriculture by answer farm...
Kisan Call Centre - To harness potential of ICT in Agriculture by answer farm...Kisan Call Centre - To harness potential of ICT in Agriculture by answer farm...
Kisan Call Centre - To harness potential of ICT in Agriculture by answer farm...
 
Employee wellbeing at the workplace.pptx
Employee wellbeing at the workplace.pptxEmployee wellbeing at the workplace.pptx
Employee wellbeing at the workplace.pptx
 
Painted Grey Ware.pptx, PGW Culture of India
Painted Grey Ware.pptx, PGW Culture of IndiaPainted Grey Ware.pptx, PGW Culture of India
Painted Grey Ware.pptx, PGW Culture of India
 
internship ppt on smartinternz platform as salesforce developer
internship ppt on smartinternz platform as salesforce developerinternship ppt on smartinternz platform as salesforce developer
internship ppt on smartinternz platform as salesforce developer
 
Software Engineering Methodologies (overview)
Software Engineering Methodologies (overview)Software Engineering Methodologies (overview)
Software Engineering Methodologies (overview)
 
The Most Excellent Way | 1 Corinthians 13
The Most Excellent Way | 1 Corinthians 13The Most Excellent Way | 1 Corinthians 13
The Most Excellent Way | 1 Corinthians 13
 
CARE OF CHILD IN INCUBATOR..........pptx
CARE OF CHILD IN INCUBATOR..........pptxCARE OF CHILD IN INCUBATOR..........pptx
CARE OF CHILD IN INCUBATOR..........pptx
 
Pharmacognosy Flower 3. Compositae 2023.pdf
Pharmacognosy Flower 3. Compositae 2023.pdfPharmacognosy Flower 3. Compositae 2023.pdf
Pharmacognosy Flower 3. Compositae 2023.pdf
 
POINT- BIOCHEMISTRY SEM 2 ENZYMES UNIT 5.pptx
POINT- BIOCHEMISTRY SEM 2 ENZYMES UNIT 5.pptxPOINT- BIOCHEMISTRY SEM 2 ENZYMES UNIT 5.pptx
POINT- BIOCHEMISTRY SEM 2 ENZYMES UNIT 5.pptx
 
Staff of Color (SOC) Retention Efforts DDSD
Staff of Color (SOC) Retention Efforts DDSDStaff of Color (SOC) Retention Efforts DDSD
Staff of Color (SOC) Retention Efforts DDSD
 
Class 11 Legal Studies Ch-1 Concept of State .pdf
Class 11 Legal Studies Ch-1 Concept of State .pdfClass 11 Legal Studies Ch-1 Concept of State .pdf
Class 11 Legal Studies Ch-1 Concept of State .pdf
 
Science lesson Moon for 4th quarter lesson
Science lesson Moon for 4th quarter lessonScience lesson Moon for 4th quarter lesson
Science lesson Moon for 4th quarter lesson
 

Hygien 165 krok 2

  • 1. According to the report of water quality control, drinking city water has the following characteristics: turbidity - 1,5 mg/m3, odour - 3 points, metallic taste - 2 points, pale yellow colour, colour index - 20o, temperature - 12o. Which of these factors doesn't comply with hygienic requirements? Select one: a. Taste b. Turbidity c. Colour index d. Odour e. Temperature Feedback The correct answer is: Odour Question 2 Not answered Mark 0.00 out of 1.00 Flag question Question text The air of a foundry worker's working zone contains condensation aerosol with dust particles sized 2 nm (90%), 2-5 nm (2%), over 5 nm (6%), below 2 nm (about 2%). Characterize the dust dispersivity: Select one: a. Mist b. Fine-dispersed c. Median-dispersed d. Coarsely dispersed e. Ultrafine-dispersed Feedback The correct answer is: Fine-dispersed Question 3 Not answered Mark 0.00 out of 1.00
  • 2. Flag question Question text A district doctor keeps the record of reconvalescents after infectious diseases, people who are disposed to frequent and long-lasting diseases, patients with chronic pathologies. What category of patients should belong to the III health group? Select one: a. People disposed to frequent and long-lasting diseases b. Reconvalescents after infectious diseases and patients with chronic pathologies c. People with chronic pathologies and disposed to frequent and long-lasting diseases d. All above mentioned categories e. People with chronic diseases Feedback The correct answer is: People with chronic diseases Question 4 Not answered Mark 0.00 out of 1.00 Flag question Question text To study physical development of children and adolescents, anthropometric studies are widely used. Choose a physiometric method of study from the below given. Select one: a. Determination of vital capacity of lungs b. Measurement of growth c. Determination of vertebra form d. Determination of thorax form e. Determination of body weight
  • 3. Feedback The correct answer is: Determination of vital capacity of lungs Question 5 Not answered Mark 0.00 out of 1.00 Flag question Question text As a result of prophylactic medical examination a 35 year old woman was diagnosed with alimentary and constitutive obesity of the III degree. It is known from her anamnesis that the patient doesn't observe rules of rational nutrition: she often overeats, the last food intake is usually 10-15 minutes before going to bed, prefers fattening and rich in carbohydrates food. What is the main alimentary risk factor of obesity development? Select one: a. Excess of fats b. Excess of carbohydrates c. Energetic unprofitableness of nutrition d. Violation of dietary pattern e. Lack of cellulose Feedback The correct answer is: Energetic unprofitableness of nutrition Question 6 Not answered Mark 0.00 out of 1.00 Flag question Question text Some of the population of a city district have uneven teeth color. The individuals have white spots, transverse brown stripes on the incisors. Occurrence of these symptoms is associated with the quality of drinking water from a deep well. Which of the following components of water can be the cause of the disease? Select one:
  • 4. a. Fe b. Mg c. J d. Ca e. F Feedback The correct answer is: F Question 7 Not answered Mark 0.00 out of 1.00 Flag question Question text The results of 5 year monitoring allowed to estimate the level of environmental influence upon health indices of popultaion. What statistic method should be chosen? Select one: a. Calculation of regression coefficient b. Calculation of coefficient of difference validity c. Calculation of dynamic indices d. Calculation of conformity coefficient e. Calculation of correlation coefficient Feedback The correct answer is: Calculation of correlation coefficient Question 8 Not answered Mark 0.00 out of 1.00 Flag question
  • 5. Question text Researchers studied disease incidence of influenza and acute respiratory viral infection within the last 5 years. What kind of graphic presentation should be used for the best visualization of this data? Select one: a. Linear diagram b. Histogram c. Radial diagram d. Bar diagram e. Pie diagram Feedback The correct answer is: Linear diagram Question 9 Not answered Mark 0.00 out of 1.00 Flag question Question text A student lives in the modern house in the flat with a complete set of sanitary equipment (WC, bath, shower, local water heater). How much water consumption has he got? Select one: a. 500 -600 L/day b. 300-400 L/day c. 50-100 L/day d. 160-200 L/day e. 10-15 L/day Feedback The correct answer is: 160-200 L/day Question 10 Not answered Mark 0.00 out of 1.00
  • 6. Flag question Question text A 30-year-old patient with complaints of occipital headache, disturbed sleep with nightmares came to a policlinic. BP was 150/95 mm Hg. He was diagnosed with hypertensic crisis. The patient should be registered in the following dispensary group for arterial hypertension surveillance: Select one: a. In the fifth b. In the fourth c. In the first d. In the second e. In the third Feedback The correct answer is: In the second Question 11 Not answered Mark 0.00 out of 1.00 Flag question Question text A young patient who came to a policlinic was diagnosed with the 1 stage of hypertension. How often should he undergo the medical check-up? Select one: a. Twice a year b. Once a year c. 3 times a year d. 5 times a year e. 4 times a year
  • 7. Feedback The correct answer is: Twice a year Question 12 Not answered Mark 0.00 out of 1.00 Flag question Question text Study of morbidity rate in a city N revealed that population of different administrative districts differed in age structure. What statistic method allows to eliminate influence of this factor upon morbidity indices? Select one: a. Correlative regressive analysis b. Wilcoxon's t-criterion c. Analysis of dynamic series d. Standardization e. Calculation of average values Feedback The correct answer is: Standardization Question 13 Not answered Mark 0.00 out of 1.00 Flag question Question text A driver had been fixing a car in a closed garage and afterwards complained about headache, dizziness, nausea, muscle asthenia, sleepiness. Objectively: pulse and respiratory rate elevation, excitement, hypertension, delirium of persecution. What is the most likely diagnosis? Select one: a. Asthenovegetative syndrome b. Hypertensive crisis
  • 8. c. Intoxication with carbon oxide d. Intoxication with ethyl gasoline e. Posttraumatic encephalopathy Feedback The correct answer is: Intoxication with carbon oxide Question 14 Not answered Mark 0.00 out of 1.00 Flag question Question text A factory's sectorial doctor selects a group of persons who often fall ill for thorough monitoring. At the same time he takes into consideration the number of etiologically related cases with temporary disability in each of the employees over the last year. An employee falls into this group if the number of sickness cases is: Select one: a. 3 or more b. 2 or more c. 6 or more d. 1 or more e. 4 or more Feedback The correct answer is: 4 or more Question 15 Not answered Mark 0.00 out of 1.00 Flag question Question text
  • 9. A city's population is 400000 inhabitants in 2005 there were registered 5600 deaths, including 3300 cases caused by cardiovascular diseases, 730 - by tumours. Which of the following indicators allows to characterize the share of the circulatory system diseases as the cause of death in the city? Select one: a. Ratio b. Intensive index c. Demonstrativeness index d. Index of relative intensity e. Extensive index Feedback The correct answer is: Extensive index Question 16 Not answered Mark 0.00 out of 1.00 Flag question Question text An outpatient hospital made record of 11600 diseases within one year. Among them influenza and ARD make up 5800, circulatory system diseases - 3480, digestion diseases - 1300, other diseases - 1020. What relative index can be calculated according this data? Select one: a. Visualization b. Intensive c. Extensive d. Correlation Feedback The correct answer is: Extensive Question 17 Not answered Mark 0.00 out of 1.00
  • 10. Flag question Question text A patient consulted a doctor about acure respiratory viral infection. The patient was acknowledged to be off work. The doctor issued him a medical certificate for 5 days. The patient is not recovering. What measures should the doctor take in order to legalize the further disability of patient? Select one: a. To prolong the medical certificate together with department superintendent b. To send the patient to the medical consultative commission c. To prolong the medical certificate at his own discretion but no more than for 6 days in total d. To send the patient to the medical social expert comission e. To prolong the medical certificate at his own discretion but no more than for 10 days in total Feedback The correct answer is: To prolong the medical certificate at his own discretion but no more than for 10 days in total Question 18 Not answered Mark 0.00 out of 1.00 Flag question Question text What information gathering method is preferable to study housing conditions of medical students during training period? Select one: a. Materials selection b. Directed selection method c. Interviewing d. Statistical e. Questionaire
  • 11. Feedback The correct answer is: Questionaire Question 19 Not answered Mark 0.00 out of 1.00 Flag question Question text A pupil of the 8th form after trauma has acute atrophy of the left arm muscles, tonus of which is distinctly decreased, active movements are only in the left joint, pupil's foot is deformed. Function of support of the left leg is absent, support function of the right leg is preserved. The boy wears an orthopedic footwear. What group of physical training does the boy belong to? Select one: a. Additional b. Basic c. Preparatory d. Special e. Other Feedback The correct answer is: Special Question 20 Not answered Mark 0.00 out of 1.00 Flag question Question text During the medical examination a port crane operator complained of dizziness, nausea, sense of pressure against tympanic membranes, tremor, dyspnoea, cough. He works aloft, the work is connected with emotional stress. Workers are affected by vibration (general and local), noise,
  • 12. ultrasound, microclimate that warms in summer and cools in winter. What factor are the worker's complaints connected with? Select one: a. Intensity of work b. Altitude work c. Vibration d. Noise e. Infrasound Feedback The correct answer is: Infrasound Question 21 Not answered Mark 0.00 out of 1.00 Flag question Question text In terms of megacalorie (1000 kcal = 4184 kJ) the ration of an adult includes 30 g of proteins, 37 g of fats, 137 g of carbohydrates, 15 mg of vitamin C, 0,6 mg of thiamine (vitamin B1). The ration is UNBALANCED as to the contents of: Select one: a. Fats b. Proteins c. Carbohydrates d. Vitamin C e. Thiamine Feedback The correct answer is: Vitamin C Question 22 Not answered Mark 0.00 out of 1.00
  • 13. Flag question Question text Administration of a plant producing red lead paint intends to form a group of medical specialists for periodical medical examinations. What specialist must be obligatory included into this group? Select one: a. Gynaecologist b. Neuropathologist c. Dermatologist d. Psychiatrist e. Otolaryngologyst Feedback The correct answer is: Neuropathologist Question 23 Not answered Mark 0.00 out of 1.00 Flag question Question text There were registered 500 cases of urolithiasis per 10000 inhabitants. What kind of statictical indices is presented? Select one: a. Index of visualization b. Index of compliance c. Prevalence rate d. Correlation coefficient e. Incidence rate
  • 14. Feedback The correct answer is: Prevalence rate Question 24 Not answered Mark 0.00 out of 1.00 Flag question Question text Chief physician of a polyclinic encharged a district doctor with a task to determine the pathological prevalence of disease N in his district. What document allows to estimate the disease prevalence in the population of a medical district? Select one: a. Statistic coupons (+) b. Statistic coupons (+) and (-) c. Statistic coupons (-) d. Prophylactic examinations register e. Vouchers for medical appointments Feedback The correct answer is: Prophylactic examinations register Question 25 Not answered Mark 0.00 out of 1.00 Flag question Question text Workers of fishery are subjected to low temperatures of the air (from 5 till 150C). Diseases of what organs and systems are the most frequent among workers of such enterprises? Select one: a. Respiratory system b. Liver
  • 15. c. Gastrointestinal tract d. Cardiovascular system e. Blood Feedback The correct answer is: Respiratory system Question 26 Not answered Mark 0.00 out of 1.00 Flag question Question text What juice should be included in a complex drug and dietary therapy for patients with gastric ulcer or duodenal ulcer and increased gastric juice acidity in order to accelerate the ulcer healing? Select one: a. Cabbage, cabbage and carrot b. Apple, birch and apple c. Celery, parsley d. Pumpkin e. Potato, potato and carrot Feedback The correct answer is: Potato, potato and carrot Question 27 Not answered Mark 0.00 out of 1.00 Flag question Question text
  • 16. A patient complained about problems with pain and tactile sensitivity, pain in the nail bones at the end of the working day. He works at a plant with mechanical devices. What pathology can be suspected? Select one: a. Hypovitaminosis of B1 b. Caisson disease c. Overwork symptoms d. Vibration disease e. Noise disease Feedback The correct answer is: Vibration disease Question 28 Not answered Mark 0.00 out of 1.00 Flag question Question text A children's health camp received a party of tinned food. External examination of the tins revealed that they had deep dents, could be easily concaved when pressed and wouldn't immediately return to the initial state; rust was absent; the tins were greased with inedible fat. Specify the bloat type: Select one: a. Combined b. Chemical c. Physical d. Physicochemical e. Biological Feedback The correct answer is: Physical Question 29 Not answered Mark 0.00 out of 1.00
  • 17. Flag question Question text Preventive examination of an 11 year old boy helped to determine his habitus type. It was established that the child's shoulders were deviated and brought forward, with forward flexion of head, the thorax was flattened, abdomen was convex. The child's backbone had signs of deepened cervical and lumbar curvatures. What habitus is it? Select one: a. Lordosis b. Normal c. Round-shouldered d. Corrected e. Kyphosis Feedback The correct answer is: Kyphosis Question 30 Not answered Mark 0.00 out of 1.00 Flag question Question text The total area of a ward at the therapeutical department is 28 m2. What is the maximum number of beds that can be exploited in this ward? Select one: a. 5 b. 2 c. 4 d. 1 e. 3
  • 18. Feedback The correct answer is: 4 Question 31 Not answered Mark 0.00 out of 1.00 Flag question Question text Working conditions of a building company worker are characterized by cooling microclimate effect, silica-containing dust, caustic alkali (quicklime) and noise. What medical expert should be the chief of the commission that periodically examines the workers of the mentioned category? Select one: a. Neurologist b. Therapeutist c. Ophthalmologist d. Dermatologist e. Otolaryngologist Feedback The correct answer is: Therapeutist Question 32 Not answered Mark 0.00 out of 1.00 Flag question Question text In order to improve organism tolerance of boarding-school pupils a doctor developed a program. The program is based upon the following principles: graduality, consistency, individuality, coomplexity. What of the main principles of organism tempering wasn't taken into account? Select one:
  • 19. a. Increase of resistance b. Increase of influence intensity c. Increase of influence force d. Systematicness e. Autodefense increase Feedback The correct answer is: Systematicness Question 33 Not answered Mark 0.00 out of 1.00 Flag question Question text At year-end hospital administration has obtained the following data: annual number of treated patients and average annual number of beds used for patients' treatment. What index of hospital work can be calculated on the base of this data? Select one: a. Average annual bed occupancy b. Bed resources of the hospital c. Average duration of patients' presence in the hospital d. Average bed idle time e. Bed turnover Feedback The correct answer is: Bed turnover Question 34 Not answered Mark 0.00 out of 1.00
  • 20. Flag question Question text On medical observation a doctor identified girl (162 cm tall and 59 kg weight) who complained loss of ability to see surrounding objects clearly in the evening. On examination: dry skin, hyperkeratosis. Her daily ration includes the following vitamines: vitamine А– 0,5 mg, vit.В1– 2,0 mg, vit.В2– 2,5 mg, vit.В6– 2 mg, vit.С– 70 mg. What is the hypovitaminosis type? Select one: a. B2-hypovitaminosis b. B6-hypovitaminosis c. A-hypovitaminosis d. B1-hypovitaminosis e. C-hypovitaminosis Feedback The correct answer is: A-hypovitaminosis Question 35 Not answered Mark 0.00 out of 1.00 Flag question Question text 200 patients suffering from essential hypertension were examined in order to obtain data about patients' arterial pressure and age. What statistic value should be applied in order to measure relation between these characteristics? Select one: a. Student's coefficient b. Correlation coefficient c. Representation error d. Sygmal deviation e. Coefficient of variation
  • 21. Feedback The correct answer is: Correlation coefficient Question 36 Not answered Mark 0.00 out of 1.00 Flag question Question text The major repair of a hospital included renewal of colour design of hospital premises because it is of great psychological and aesthetical importance; and so the walls of patient wards will be painted under consideration of: Select one: a. Wall reflection coefficient b. Hospital profile c. Creation of cozy atmosphere d. Windows orientation e. Diseases of patients who will be staying in these wards Feedback The correct answer is: Windows orientation Question 37 Not answered Mark 0.00 out of 1.00 Flag question Question text In a city with population 400000 people 5600 fatal cases were recorded, including 3300 cases because of blood circulation diseases, 730 - because of tumors. What index will allow to characterize mortality from blood circulation diseases in this city? Select one:
  • 22. a. Relative intensity index b. Visuality index c. Correlation index d. Extensive index e. Intensive index Feedback The correct answer is: Intensive index Question 38 Not answered Mark 0.00 out of 1.00 Flag question Question text Choose a method of a graphic representation of monthly information about number of the registered cases of acute intestinal infection and their comparisons to the average monthly values, obtained for 5 previous years: Select one: a. The figured diagram b. The curvilinear disgram c. The radial diagram d. The linear diagram e. The sector diagram Feedback The correct answer is: The linear diagram Question 39 Not answered Mark 0.00 out of 1.00
  • 23. Flag question Question text A 38 year old man was admitted to a hospital from his working place on July 19 because of hip fracture. He was invalid till November 19. Requires prolongation of treatment. Who decides on the issue of further temporary invalidity? Select one: a. DCC b. Regional MSEC c. The head physician of a polyclinic d. Specialized (traumatologic) MSEC e. Interregional general MSEC Feedback The correct answer is: Specialized (traumatologic) MSEC Question 40 Not answered Mark 0.00 out of 1.00 Flag question Question text A patient who has been consuming refined foodstuffs for a long time complains about headache, fatiguability, depression, insomnia, irritability. Objectively: muscle asthenia, pain and cramps in the gastrocnemius muscles, during walking the patient lands onto his heel first, then on the external edge of foot. Cardiovascular system exhibits tachycardia, hypoxia, dystrophic changes of myocardium. There are also gastrointestinal disorders. What is the most likely diagnosis? Select one: a. Hypovitaminosis B6 b. Hypovitaminosis B12 c. Hypovitaminosis B15 d. Hypovitaminosis B2
  • 24. e. Hypovitaminosis B1 Feedback The correct answer is: Hypovitaminosis B1 Question 41 Not answered Mark 0.00 out of 1.00 Flag question Question text Deputy of chief medical officer carried out a study of morbidity rate for population which had been served at the polyclinics within the last 5 years. What statistical values can help in calculation of morbidity rates? Select one: a. Average values b. Dynamic series c. Absolute values d. Relative values e. Standard values Feedback The correct answer is: Relative values Question 42 Not answered Mark 0.00 out of 1.00 Flag question Question text A selective population research study was aimed at exploring the effect of air emissions from a metallurgical plant on the obstructive bronchitis morbidity in a city. The calculated correlation coefficient was +0,79. Evaluate the strength and direction of the relationship:
  • 25. Select one: a. Inverse, strong b. Inverse, average c. Direct, strong d. Direct, average Feedback The correct answer is: Direct, strong Question 43 Not answered Mark 0.00 out of 1.00 Flag question Question text At first appointment with an obstetrician-gynaecologist a pregnant woman is referred to other medical specialists. She must be obligatory examined by the following specialists: Select one: a. Dentist and phthisiatrician b. ENT and ophthalmologist c. Therapeutist and dentist d. Therapeutist and endocrinologist e. Dentist and cardiologist Feedback The correct answer is: Therapeutist and dentist Question 44 Not answered Mark 0.00 out of 1.00 Flag question
  • 26. Question text During coal extraction in a mine the concentration of coal dust in the working area is 450 mg/m3 (MPC is 10 mg/m3). What occupational respiratory disease may develop in miners? Select one: a. Anthracosis b. Allergic nasopharyngitis c. Talcosis d. Siderosis e. Byssinosis Feedback The correct answer is: Anthracosis Question 45 Not answered Mark 0.00 out of 1.00 Flag question Question text Over a current year among workers of an institution 10% haven't been ill a single time, 30% have been ill once, 15% - twice, 5% - 4 times, the rest - 5 and more times. What is the percentage of workers relating to the I health group? Select one: a. 60% b. 10% c. 22% d. 55% e. 40% Feedback The correct answer is: 55% Question 46 Not answered Mark 0.00 out of 1.00
  • 27. Flag question Question text A military unit stopped for 3-day's rest in an inhabited locality after a long march. The sanitary- epidemiological reconnaissance found several water sources. It is necessary to choose the source complying with the hygienic standards for drinking water in the field conditions: Select one: a. Melt snow water b. Artesian well water c. River water d. Spring water e. Rain water Feedback The correct answer is: Artesian well water Question 47 Not answered Mark 0.00 out of 1.00 Flag question Question text Analysis of organization of medical care in a regional centre has shown that every year about 12% of patients receive inpatient care for diseases that don't require round-the-clock monitoring and intensive care. What are the most appropriate organizational changes required to address this problem? Select one: a. Development of medical care forms replacing the in-patient care b. Development of primary care c. Changes to the statute of outpatient clinics d. Upgrading of hospital facilities e. Restructuring of specialized care
  • 28. Feedback The correct answer is: Development of medical care forms replacing the in-patient care Question 48 Not answered Mark 0.00 out of 1.00 Flag question Question text A heat station working on solid fuel is located in a residential district. On cloudy foggy days in december there was an increase in diseases with upper airway affection and signs of general intoxication. There were also mortal cases among the elderly people. What is the most likely factor that provoked toxic effect? Select one: a. Suspended materials b. Low air temperature c. High air humidity d. Temperature gradient e. Calm Feedback The correct answer is: Suspended materials Question 49 Not answered Mark 0.00 out of 1.00 Flag question Question text District physician was charged with plan drafting concerning medical and preventive measures among the population in the area he is assigned to. What measures must he include in this plan as regards primary prevention of illness?
  • 29. Select one: a. Measures to improve patients' life conditions b. Prevention of disease onset c. Measures to increase patients' life quality d. Prevention of disease complications e. Referral of patients to sanatorium Feedback The correct answer is: Prevention of disease onset Question 50 Not answered Mark 0.00 out of 1.00 Flag question Question text A worker diagnosed with "acute dysentery" was sent to the infectious department by a doctor of aid post. What document should be used for registration of this disease? Select one: a. Outpatient's card b. Statistic card of the patient who left in-patient hospital c. Statistic coupon for registration of final diagnoses d. Urgent report on infectious disease e. Inpatient's card Feedback The correct answer is: Urgent report on infectious disease Question 51 Not answered Mark 0.00 out of 1.00
  • 30. Flag question Question text In one of the surgical departments the quality assurance testing of sterilization of surgical instruments was performed. After an instrument had been treated with 1% phenolphthalein, the solution turned pink. This indicates that the instrument has: Select one: a. Residual blood b. Drugs residues c. Residual tissue d. Synthetic detergent residues e. Disinfectant residues Feedback The correct answer is: Synthetic detergent residues Question 52 Not answered Mark 0.00 out of 1.00 Flag question Question text A department chief of an in-patient hospital is going to inspect resident doctors as to observation of medical-technological standards of patient service. What documentation should be checked for this purpose? Select one: a. Annual report of a patient care institution b. Registry of operative interventions c. Health cards of in-patients d. Treatment sheets e. Statistic cards of discharged patients
  • 31. Feedback The correct answer is: Health cards of in-patients Question 53 Not answered Mark 0.00 out of 1.00 Flag question Question text While making sanitary examination of burn unit for adults it was stated that wards for 4 persons are of 28 m2 square. What should be the minimum ward area in this unit? Select one: a. 52 м2 b. 28 м2 c. 30 м2 d. 40 м2 e. 24 м2 Feedback The correct answer is: 40 м2 Question 54 Not answered Mark 0.00 out of 1.00 Flag question Question text A plot of land with total area of 2,0 hectare was intended for building of a hospital. The maximal capacity of the hospital will be: Select one: a. 200 beds
  • 32. b. Over 1000 beds c. 100 beds d. 400 beds e. 800 beds Feedback The correct answer is: 100 beds Question 55 Not answered Mark 0.00 out of 1.00 Flag question Question text A 59-year-old male patient with essential hypertension of stage II is registered with the dispensary department of a polyclinic. The patient regularly takes ACE inhibitors and calcium antagonists. How often should a therapeutist examine this patient (except for exacerbation periods)? Select one: a. Once a year b. Every 6 months c. Every 4 months d. Every 9 months e. Every 3 months Feedback The correct answer is: Every 3 months Question 56 Not answered Mark 0.00 out of 1.00 Flag question
  • 33. Question text A local doctor has to prepare a report about the health condition of the population of his region. What medical indexes of population health condition should he use? Select one: a. Average treatment duration, complications b. Average longevity c. Morbidity, disabilities, demographic, physical development d. Social welfare, satisfaction of life quality e. Way of life, genetic, pollution Feedback The correct answer is: Morbidity, disabilities, demographic, physical development Question 57 Not answered Mark 0.00 out of 1.00 Flag question Question text Within the structure of the region's population the share of persons aged 0 to 14 years is 25%, the share of persons aged 50 years and older is 30%. What concept most accurately describes this demographic situation? Select one: a. Cohort reproduction b. Progressive type of population age structure c. Regressive type of population age structure d. Immigration of population e. Stationary type of population age structure Feedback The correct answer is: Regressive type of population age structure Question 58 Not answered Mark 0.00 out of 1.00
  • 34. Flag question Question text A 2 year old child has been ill with acute respiratory viral infection of upper thrice a year - in February, in April and in December. How should these occurences be recorded? Select one: a. It is necessary to fill in 1 statistic talon signed (-) b. It is necessary to fill in 1 statistic talon signed (+) and 2 statistic talons signed (-) c. It is necessary to fill in 3 statistic talons signed (+) d. It is necessary to fill in 1 statistic talon signed (+) e. It is necessary to fill in 3 statistic talons signed (-) Feedback The correct answer is: It is necessary to fill in 3 statistic talons signed (+) Question 59 Not answered Mark 0.00 out of 1.00 Flag question Question text In treatment and prevention establishments, regardless of their organisational and proprietary form, the rights of patients should be observed. Which of these rights is the most significant? Select one: a. The right to the protection of the patient's interests b. The right to the free choice c. The right to the information d. The right to be heard e. The right to the protection from incompetence
  • 35. Feedback The correct answer is: The right to the protection of the patient's interests Question 60 Not answered Mark 0.00 out of 1.00 Flag question Question text A factory worker has ARD complicated by acute bronchitis. He receives treatment in the outpatient setting. The attending doctor has issued him a medical certificate for 5 days and then extended its duration by 5 more days. Patient can't get down to work because of his health status. Who should extend the duration of medical certificate for this patient? Select one: a. A department chief b. Deputy medical superintendent in charge of medical treatment c. Medical advisory commission d. Medical superintendent e. Deputy medical superintendent in charge of temporary disability examination Feedback The correct answer is: A department chief Question 61 Not answered Mark 0.00 out of 1.00 Flag question Question text Point out the unit for statistical observation for the determination of blood sugar level influence on a wound surface healing during postoperative period. Select one: a. The patient who has a wound surface b. The patient in a postoperative period
  • 36. c. Blood test d. Blood sugar level e. The patient who was discharged to outpatient treatment Feedback The correct answer is: The patient in a postoperative period Question 62 Not answered Mark 0.00 out of 1.00 Flag question Question text A district pediarician has carried out infant mortality rate analysis in his area. What data has been used? Select one: a. Mortality of children under 1 y.o., natimortality b. Mortality of children under 1 y.o. structured by age, sex, causes c. Mortality of district newborn d. Hospital mortality of children, structured by age e. Mortality of district adolescents Feedback The correct answer is: Mortality of children under 1 y.o. structured by age, sex, causes Question 63 Not answered Mark 0.00 out of 1.00 Flag question Question text
  • 37. The Carpathian region is characterized by constant high humidity of atmospheric air (over 80%). Inhabitants of this region feel severe cold in corresponding season at a medium low temperature. It's caused by heat emission by: Select one: a. Convection b. Conduction c. Vaporization d. Radiation Feedback The correct answer is: Convection Question 64 Not answered Mark 0.00 out of 1.00 Flag question Question text A district doctor was commisioned with a task to work out a plan of treatment-and-prophylaxis actions for the population of his district. What actions of secendary prophylaxis must he include into this plan? Select one: a. Rehabilitation actions b. Prevention of disease complications c. Elimination of disease causes d. Improvement of population's living conditions e. Disease prevention Feedback The correct answer is: Prevention of disease complications Question 65 Not answered Mark 0.00 out of 1.00
  • 38. Flag question Question text During the periodic medical examination an assembly fitter (works on soldering details) didn't report any health problems. Closer examination revealed signs of asthenic-vegetative syndrome. Blood included red blood cells with basophilic aggregations and a somewhat higher number of reticulocytes, urine had a high concentration of delta-aminolevulinic acid. The complex of symptoms indicates the initial stage of chronic intoxication with: Select one: a. Ethanol b. Lead c. Mercury d. Manganese e. Tin Feedback The correct answer is: Lead Question 66 Not answered Mark 0.00 out of 1.00 Flag question Question text An outbreak of food poisoning was recorded in an urban settlement. The illness was diagnosed as botulism on the grounds of clinical presentations. What foodstuffs should be chosen for analysis in the first place in order to confirm the diagnosis? Select one: a. Pasteurized milk b. Potatoes c. Cabbage d. Boiled meat
  • 39. e. Tinned food Feedback The correct answer is: Tinned food Question 67 Not answered Mark 0.00 out of 1.00 Flag question Question text Environmental pollution is prevented by mechanical separation of nontoxic solid domestic waste. Specify the method which can be used for mechanical utilization of these wastes: Select one: a. Burning as power-plant fuel b. Waste neutralization in biothermal boxes c. Compressing of wastes into building blocks d. Burial of wastes e. Hydrolysis Feedback The correct answer is: Compressing of wastes into building blocks Question 68 Not answered Mark 0.00 out of 1.00 Flag question Question text A 50-year-old male suburbanite underwent treatment in rural outpatient clinic for pneumonia. The treatment didn't have effect and the disease got complicated by exudative pleuritis. What prevention and treatment facility should the patient be referred to for further aid? Select one:
  • 40. a. Tuberculosis dispensary b. Regional hospital c. Central district hospital d. Phthisio-pulmonological dispensary e. Municipal hospital Feedback The correct answer is: Central district hospital Question 69 Not answered Mark 0.00 out of 1.00 Flag question Question text There is a dynamic growth of number of congenital abnormalities such as central paralysis, newborns blindness, idiocy among the population that lives near to pesticides production enterprise. Compounds of which pollutant can cause the development of this pathology? Select one: a. Strontium b. Chrome c. Cadmium d. Mercury e. Iron Feedback The correct answer is: Mercury Question 70 Not answered Mark 0.00 out of 1.00
  • 41. Flag question Question text A mother who is on partially paid maternity leave got sick and was hospitalized. What document is to be issued to a working father who will be taking care of a child during his mother's illness? Select one: a. Certificate of mother's illness b. Free-form certificate c. Certificate of child's care necessity d. Sick-leave e. Extract from the medical card of out- or in-patient Feedback The correct answer is: Sick-leave Question 71 Not answered Mark 0.00 out of 1.00 Flag question Question text Poorly refined wastes of an industrial plant are usually thrown into the river that supplies drinking water. It causes perishing of some microorganisms, disturbs processes of water self-purification and worsens its quality that can have negative influence upon people's health. How is this effect of environmental factors called? Select one: a. Combined b. Associated c. Indirect d. Complex e. Direct
  • 42. Feedback The correct answer is: Indirect Question 72 Not answered Mark 0.00 out of 1.00 Flag question Question text District doctor of rural medical department was called to a 42-year-old patient. While examining the patient, the doctor suspected the dysentery. What document must the doctor issue? Select one: a. Report addressed to Head of the village b. Statistical coupon of final (precise) diagnosis c. Urgent notification of infectious disease d. Abstract of outpatient medical card e. Infectious disease report Feedback The correct answer is: Urgent notification of infectious disease Question 73 Not answered Mark 0.00 out of 1.00 Flag question Question text The institutions which take part in medical examinations can be prevention and treatment facilities, medical board of Ministry of Defense, medical board of Ministry of Home Affairs, medico-social expert commissions, forensic medical boards etc. What institutions are responsible for temporary disability examination? Select one:
  • 43. a. Medical boards of Ministry of Home Affairs b. Sanitary-and-prophylactic institutions c. Medico-social expert commissions d. Medical boards of Ministry of Defense e. Prevention and treatment facilities Feedback The correct answer is: Prevention and treatment facilities Question 74 Not answered Mark 0.00 out of 1.00 Flag question Question text Periodical survey of a worker of a chemicals plant revealed a malignant neoplasm on the urinary bladder. This occupational disease was the most probably caused by contact with the following industrial poison: Select one: a. Benzidine b. Vinyl chloride c. Nickel carbonyl d. Arsenic e. Asbestos Feedback The correct answer is: Benzidine Question 75 Not answered Mark 0.00 out of 1.00
  • 44. Flag question Question text The student has the following devices: Geiger counter, Ebert counter, Krotov's apparatus, Mischuk device, Ebert device. What device can he use to assess air germ pollution? Select one: a. Mischuk's device b. Geiger's counter c. Krotov's apparatus d. Ebert's device e. Ebert's counter Feedback The correct answer is: Krotov's apparatus Question 76 Not answered Mark 0.00 out of 1.00 Flag question Question text Head of a department and a trade-union group have appealed to the head of a hospital about dismissal of the senior nurse who has 17-year record of service. The facts of charge were confirmed and recognized by the nurse herself. The nurse lives with a daughter (who is divorced and unemployed) and a 9-month-old grandson. Make an administrative decision: Select one: a. To issue the sick leave b. To discharge the worker, i.e. to satisfy demands of the collective c. To continue the worker in office with a warning of dismissal in case of repeated violation of labor discipline d. To embark other officials or public organizations with this problem
  • 45. Feedback The correct answer is: To continue the worker in office with a warning of dismissal in case of repeated violation of labor discipline Question 77 Not answered Mark 0.00 out of 1.00 Flag question Question text During the medical examination at school the schoolchildren had to undergo plantography. After the analysis of footprints platypodia was found in 30% of pupils. What is the percentage of the flatfoot isthmus? Select one: a. 30% b. 45% c. 65% d. 55% e. 50% Feedback The correct answer is: 65% Question 78 Not answered Mark 0.00 out of 1.00 Flag question Question text A patient undergoes inpatient treatment with the diagnosis of acute pancreatitis. To spare pancreas as much as possible the doctor prescribed for him starvation for 1-3 days. What products is the patient allowed to eat during recovery period after cancelling of starvation? Select one: a. Potato and carrot mash
  • 46. b. Grape juice c. Milk d. Broth e. Boiled meat Feedback The correct answer is: Potato and carrot mash Question 79 Not answered Mark 0.00 out of 1.00 Flag question Question text A 48-year-old male in-patient undergoes treatment for essential hypertension of II-B stage. It is known from history that he works in a design engineering office. His job involves neuro-emotional stress. Which of these foodstuffs do not stimulate the central nervous system and can be recommended for the patient? Select one: a. Whole milk b. Carbonated beverages c. Meat broths d. Vegetable broths e. Mushroom broths Feedback The correct answer is: Whole milk Question 80 Not answered Mark 0.00 out of 1.00 Flag question
  • 47. Question text The amount of ultraviolet radiation dose was measured in minutes. What device was applied for measurement of the biodose? Select one: a. UV-meter b. Radiometer c. Actinometer d. Catathermometer e. Gorbachev's biodosimeter Feedback The correct answer is: Gorbachev's biodosimeter Question 81 Not answered Mark 0.00 out of 1.00 Flag question Question text Among the inhabitants of a workmen's settlement located near an industrial plant the cases of nervous and endocrine system diseases as well as renal diseases became more frequent. Blood analyses revealed a decrease in sulfhydryl groups. These pathologies might have been caused by the following substance released into the environment: Select one: a. Chromium b. Mercury c. Cadmium d. Boron e. Lead Feedback The correct answer is: Mercury Question 82 Not answered
  • 48. Mark 0.00 out of 1.00 Flag question Question text An employee has been diseased for 4 months, further treatment is necessary, the patient is incapacitated. Who is authorized to provide further disability examination of this patient? Select one: a. Deputy chief responsible for disability examination b. Medical and social expert board c. Physician in charge and chief of department d. Chief physician of a medical facility e. Medical consultative board Feedback The correct answer is: Medical and social expert board Question 83 Not answered Mark 0.00 out of 1.00 Flag question Question text Statistic of patients of common medical practice displays constant increase in elderly and old people number. What kind of pathology is expected to prevail in the morbidity structure of population in question? Select one: a. Acute pathology b. Chronic pathology c. Non-epidemic pathology d. Occupational pathology e. Infectious pathology
  • 49. Feedback The correct answer is: Chronic pathology Question 84 Not answered Mark 0.00 out of 1.00 Flag question Question text What is the maximum duration of medical certificate in case of tuberculosis? Select one: a. 10 months b. 2 months c. Week d. 2 weeks e. Month Feedback The correct answer is: 2 months Question 85 Not answered Mark 0.00 out of 1.00 Flag question Question text It is planned to organize a rural outpatient clinic. The patients will be able to visit the doctors of the following specialities: Select one: a. Obstetrician-gynaecologist, therapeutist b. Therapeutist, dentist, pediatrician, obstetrician-gynecologist
  • 50. c. Pediatrician, obstetrician-gynaecologist, ophthalmologist d. Pediatrician, therapeutist, ophthalmologist e. Therapeutist, pediatrician, neurologist Feedback The correct answer is: Therapeutist, dentist, pediatrician, obstetrician-gynecologist Question 86 Not answered Mark 0.00 out of 1.00 Flag question Question text A child is 6 years old. Within one year of observation he had URI that lasted 8 days. Physical state is satisfactory. Specify hi health group: Select one: a. II b. III (b) c. III (c) d. I e. III (a) Feedback The correct answer is: I Question 87 Not answered Mark 0.00 out of 1.00 Flag question Question text
  • 51. A municipal hospital reported on the number of operated patients including fatal outcomes following the operations. Which index of hospital work can be calculated on the ground of this data? Select one: a. Postoperative lethality b. Index of late hospitalization since a disease incursion c. Standardized lethality d. Total lethality Feedback The correct answer is: Postoperative lethality Question 88 Not answered Mark 0.00 out of 1.00 Flag question Question text Head of a department and a trade-union group have appealed to the head of a hospital about dismissal of the senior nurse who has 17 year record of service. The facts of charge were confirmed and recognized by the nurse herself. This nurse lives with a daughter (who is divorced and unemployed) and a 9-month-old grandson. Make an administrative decision: Select one: a. To issue the sick list b. To shift the solution of this problem on other officials or public organizations c. To discharge the worker, i.e. to satisfy demands of the collective d. To continue the worker in office with a warning of dismissal in case of repeated violation of labor discipline Feedback The correct answer is: To continue the worker in office with a warning of dismissal in case of repeated violation of labor discipline Question 89 Not answered Mark 0.00 out of 1.00
  • 52. Flag question Question text A 46-year-old patient once took part in elimination of breakdown at an atomic power plant. Currently he is being treated at an in-patient hospital. He was diagnosed with progressing vegetative insufficiency. This disease relates to the following group of ionizing radiation effects: Select one: a. Hormesis b. Somato-stochastic c. Heterosis d. Somatic e. Genetic Feedback The correct answer is: Somato-stochastic Question 90 Not answered Mark 0.00 out of 1.00 Flag question Question text Bacterial analysis of air in a living space in winter period by means of Krotov's apparatus revealed that total number of microorganisms in 1 m3 of air was 7200. What is the permissible number of microorganisms for the air to be characterized as "pure"? Select one: a. Up to 7500 b. Up to 5500 c. Up to 2500 d. Up to 4500 e. Up to 3500
  • 53. Feedback The correct answer is: Up to 4500 Question 91 Not answered Mark 0.00 out of 1.00 Flag question Question text It is planned to make complete isolation boxes in the infectious department in order to prevent nosocomial airborne infections. The boxes consist of a tambour, a ward and a lock chamber. What structure should be also included in a complete isolation box? Select one: a. Doctor's consulting room b. Patient's examination room c. Manipulation room d. Nursing room e. Bathroom unit Feedback The correct answer is: Bathroom unit Question 92 Not answered Mark 0.00 out of 1.00 Flag question Question text An employee of a private company was ill with acute respiratory viral infection. Consulted a district doctor, who determined the fact of temporary loss of working ability, but refused to issue a sick-list, arguing that the patient worked in the private and not state-owned company. Should the sick-list be issued to the employees of private companies?
  • 54. Select one: a. Issued only on condition of payment guarantee by the company's proprietor b. Issued a medical certificate of a set form c. Issued a medical certificate of a free form d. Issued only to empties of state-owned companies e. Issued regardless of company's ownership Feedback The correct answer is: Issued regardless of company's ownership Question 93 Not answered Mark 0.00 out of 1.00 Flag question Question text In a rural health district a child died in the first month of life. In order to analyze this situation it was necessary to complete expert evaluation of medical records. Which medical document was analyzed in the first place? Select one: a. Neonatal record b. Medical record of an outpatient c. Child developmental history record d. Child's medical record e. Record of vaccinations Feedback The correct answer is: Child developmental history record Question 94 Not answered Mark 0.00 out of 1.00
  • 55. Flag question Question text Examination of a 13-year-old boy reveals that his body length is 147 сm (+2), body weight - 38 kg (+1,5), circumference of chest - 72 cm (+0,2). Estimate the harmonicity of the child's physical development: Select one: a. Disharmonious b. Above the average c. Sharply disharmonious d. Harmonious e. Supernormal Feedback The correct answer is: Disharmonious Question 95 Not answered Mark 0.00 out of 1.00 Flag question Question text Study of actual diet of an adult revealed the following: proteins make up 16% of energy value of daily ration, fats - 25%, carbohydrates - 59%. Evaluate compliance of protein, fat and carbohydrate share in the energy value of daily ration with the recommended shares of these nutrients? Select one: a. Carbohydrate share is insufficicent b. Carbohydrate share is excessive c. Fat share is insufficient d. Nutrient content complies with the recommended shares of energy value e. Carbohydrate share is insufficient, there is excess of proteins
  • 56. Feedback The correct answer is: Carbohydrate share is insufficient, there is excess of proteins Question 96 Not answered Mark 0.00 out of 1.00 Flag question Question text Atmospheric air of an industrial centre is polluted with the following wastes of metallurgical plants: sulphuric, nitric, metal, carbon oxides that have negative influence upon the inhabitants' health. The effct of these hazards can be characterized as: Select one: a. Mixed b. Complex c. Adjacent d. Associated e. Combined Feedback The correct answer is: Combined Question 97 Not answered Mark 0.00 out of 1.00 Flag question Question text A 37 year old patient applied to a local therapeutist. As a result of exacerbation of chronic obstructive bronchitis the patient had been temporarily disabled for 117 days within 1 year. What tactics will be legally correct? Select one:
  • 57. a. The therapeutist should extend a medical certificate b. The patient shoul be referred to the sanatorium-and-spa treatment c. The therapeutist should issue a new medical certificate d. The patient should be referred to the medical consultation comission for extension of medical certificate e. The patient should be referred to the medicosocial expertise Feedback The correct answer is: The patient should be referred to the medicosocial expertise Question 98 Not answered Mark 0.00 out of 1.00 Flag question Question text 350 workers of a metalurgical plant had to undergo a yearly preventive examination. A territorial polyclinic carried out preventive examination of 325 workers. As a result of it, 1 worker was recognized as temporarily disabled, 15 workers underwent further rehabilitation at an after-work sanatorium, 10 workers were provided with diet meal. What index characterizing the preventive work of the polyclinic should be applied in this case? Select one: a. Frequency of case detection during examinations b. Coverage of preventive medical examinations c. Percentage of people who underwent rehabilitation at an after-work sanatorium d. Percentage of people who were provided with diet meal e. Percentage of temporarily disabled people Feedback The correct answer is: Coverage of preventive medical examinations Question 99 Not answered Mark 0.00 out of 1.00
  • 58. Flag question Question text What method is applied to establish rate of correlation between age of men and their mortality due to myocardial infarction? Select one: a. The quadrate method (Pirson) b. Correlation ratio c. The Indirect method (Student) d. Method of graduated correlation (Armler) e. Method of grade correlation (Spirman) Feedback The correct answer is: Method of grade correlation (Spirman) Question 100 Not answered Mark 0.00 out of 1.00 Flag question Question text During inspection of sanitary conditions of studying at a technical university it was necessary to evaluate the visual regimen of students, who study from 9 a.m to 3 p.m. What index of natural light will be the most informative? Select one: a. Time of the room insolation b. Presence of mixed (superolateral) light c. Natural light coefficient d. Depth of study room e. Light coefficient
  • 59. Feedback The correct answer is: Natural light coefficient Question 101 Not answered Mark 0.00 out of 1.00 Flag question Question text The head of prenatal care department intends to complete expert evaluation of compliance with medical and technological standards of pregnancy follow-up. What documents must be verified for this purpose? Select one: a. Labour and delivery record b. Neonatal record c. Medical records of outpatients d. Individual records of pregnant and postpartum women e. Prenatal records Feedback The correct answer is: Individual records of pregnant and postpartum women Question 102 Not answered Mark 0.00 out of 1.00 Flag question Question text Studying of pulmonary tuberculosis incidence provided data about patients' socioeconomic living conditions and bad habits. What method allows to estimate the impact of these factors on tuberculosis incidence? Select one: a. Calculation of correspondence index b. Calculation of regression coefficient
  • 60. c. Standardized index calculation d. Calculation of reliability coefficient e. Calculation of correlation coefficient Feedback The correct answer is: Calculation of correlation coefficient Question 103 Not answered Mark 0.00 out of 1.00 Flag question Question text A doctor of the general practice has registered the following death causes for the previous year: the first place was taken by cardiovascular diseases (60%), the second - by tumors (18%), then - traumas (8,3%) etc. What diagrams will provide the most substantial information about the registered ocurrences? Select one: a. Cartogram b. Column diagram c. Pie diagram d. Line diagram e. Circle diagram Feedback The correct answer is: Pie diagram Question 104 Not answered Mark 0.00 out of 1.00 Flag question Question text
  • 61. In a forest summer camp children have variable procedures to harden their organisms. What procedure has the most hardening power? Select one: a. Bath with hydromassage b. Morning exercises on the fresh air c. Walking on the fresh air d. Hygienic shower e. Contrast shower Feedback The correct answer is: Contrast shower Question 105 Not answered Mark 0.00 out of 1.00 Flag question Question text A 42 year old metalworker has been working at the turning machine for production of heavy large- size parts for 5 years. His work requires using of hand and pedal levers that involves considerable physical force. What means for osteoarthrosis prevention should be recommended? Select one: a. To go in for weightlifting b. To administer protein-and-vitamin diet c. To administer protein-and-carbohydrate diet d. To improve health at the Black sea coast e. To limit physical work Feedback The correct answer is: To limit physical work Question 106 Not answered Mark 0.00 out of 1.00
  • 62. Flag question Question text On physiologic-sanitary examination of railway department work it was revealed that loaders work is of III degree of difficulty. They unload vagons with sand, manually break coagulated mass by shovel and shift it. What criteria was used to evaluate work of loaders? Select one: a. Maximun load weigh which is shifted b. Time of passive observation, % to the shift duration c. Intellectual efforts d. Value of static loading for the shift e. Time of active activities, % to the shift duration Feedback The correct answer is: Maximun load weigh which is shifted Question 107 Not answered Mark 0.00 out of 1.00 Flag question Question text In an urban settlement situated on the riverbank, an outbreak of hepatitis A was registered. The disease might have water origin. This assumption can be confirmed by growth of the following values of water quality: Select one: a. Escherichia coli index b. Index of fecal coli-forms c. Oxidability d. Number of coli-phages e. Presence of benign leptospirosis pathogen
  • 63. Feedback The correct answer is: Number of coli-phages Question 108 Not answered Mark 0.00 out of 1.00 Flag question Question text An engineer-chemist at the age of 47 often fells ill with an occupational skin disease. Who makes a decision to transfer him to other job accepts? Select one: a. A head physician b. The attending physician c. MSEC d. DCC e. The chief of shop Feedback The correct answer is: DCC Question 109 Not answered Mark 0.00 out of 1.00 Flag question Question text Educational rooms are illuminated with various lighting fittings. What type of lighting fittings is the most appropriate in respect of hygienic norms? Select one: a. Combined light fittings
  • 64. b. Indirect light fittings c. Direct light fittings d. Semi-reflected light fittings e. Ambient light fittings Feedback The correct answer is: Indirect light fittings Question 110 Not answered Mark 0.00 out of 1.00 Flag question Question text A therapeutist needs to analyze adult health in the area of service. Which groups of indicators will be included into this analysis? Select one: a. Sickness rates, disability, death rates b. Birth rates, sickness rates, disability c. Demographic, sickness rates, physical development d. Sickness rates, death rates, physical development e. Demographic, sickness rates, disability Feedback The correct answer is: Demographic, sickness rates, disability Question 111 Not answered Mark 0.00 out of 1.00 Flag question Question text
  • 65. Thyreotoxicosis patient is in the two-place hospital ward of therapeutic department. The area of the ward is 18 m2, height is 3 m, ventilation rate is 2,5/h. Air temperature is 200С, relative humidity is 45%, air movement velocity is 0,3 m/sec, light coefficient is 1/5, noise level constitutes 30 dB. Make a hygienic assessment of these conditions. Select one: a. Poor lighting b. All conditions are OK c. Non-effective ventilation d. Discomfortable microclimate e. High level of noise Feedback The correct answer is: Discomfortable microclimate Question 112 Not answered Mark 0.00 out of 1.00 Flag question Question text Hygienic expertise of a sample taken from the batch of grain revealed that 2% of grains were infected with microscopic Fusarium fungi. On the ground of laboratory analyses this batch of grain should be: Select one: a. Used for forage production b. Sold without restrictions c. Used for ethanol production d. Tested for toxicity e. Destroyed Feedback The correct answer is: Sold without restrictions Question 113 Not answered Mark 0.00 out of 1.00
  • 66. Flag question Question text A 9 y.o. girl has an average height and harmonic growth development. She was ill with acute respiratory infection for five times. Define the group of her health. Select one: a. 5th group b. 2nd group c. 4th group d. 3rd group e. 1st group Feedback The correct answer is: 2nd group Question 114 Not answered Mark 0.00 out of 1.00 Flag question Question text Indicate the registration medical document for the patient, who 21.02. was addressed to the doctor with diagnosis ARVD for the first time in this year: Select one: a. The necessary registration form is not indicated b. The statistical coupon for registration of final diagnosis is not necessary c. It is necessary to fill in the emergency notice on a case of a contagion d. The statistical coupon is to be filled in, but a sign (+) is not necessary to be put in e. The statistical coupon is to be filled in and it is necessary to deliver on a sign (+)
  • 67. Feedback The correct answer is: The statistical coupon is to be filled in and it is necessary to deliver on a sign (+) Question 115 Not answered Mark 0.00 out of 1.00 Flag question Question text A teacher of a secondary school was diagnosed with pulmonary tuberculosis. What is the maximum duration of his medical certificate? Select one: a. A month b. Two months c. Four months d. Ten months e. Five months Feedback The correct answer is: Ten months Question 116 Not answered Mark 0.00 out of 1.00 Flag question Question text Estimation of community health level involved analysis of a report on diseases registered among the population of district under charge (reporting form 12). What index is calculated on the grounds of this report? Select one: a. Common morbidity rate b. Index of morbidity with temporary disability
  • 68. c. Index of pathological affection d. Index of basic non-epidemic morbidity e. Index of hospitalized morbidity Feedback The correct answer is: Common morbidity rate Question 117 Not answered Mark 0.00 out of 1.00 Flag question Question text A 46-year-old patient was issued a 10-day sick list because of exacerbation of chronic cholecystitis. The patient's general condition got better, but the clinical manifestations of the disease are still present. What authority is entitled to extend the sick list? Select one: a. Family doctor b. Deputy head doctor for medical-labour expertise c. Medical Expert Commission d. Head doctor e. Deputy head doctor for terapeutic management Feedback The correct answer is: Medical Expert Commission Question 118 Not answered Mark 0.00 out of 1.00 Flag question Question text
  • 69. Chief district pediatrician has to carry out analysis of infant mortality rate. What should he take as a unit of the observation? Select one: a. Child death case during first 7 days of life b. Child death case at the age up to the first month c. Child death case at the age up to 1 year d. Child death case on labor e. Child death case after 28 days of life Feedback The correct answer is: Child death case at the age up to 1 year Question 119 Not answered Mark 0.00 out of 1.00 Flag question Question text The district pediatrician is charged with the analysis of infant mortality. What is taken for the unit of observation in infant mortality investigation? Select one: a. A baby dead at the age over 28 days b. A baby dead at the age up to 6 days c. A baby dead at the age up to 12 months d. A baby dead at the age up to 1 months e. A baby dead at birth Feedback The correct answer is: A baby dead at the age up to 12 months Question 120 Not answered Mark 0.00 out of 1.00
  • 70. Flag question Question text An emergency situation at a chemical plant caused acute occupational intoxication. A doctor who revealed the case of "acute occupational disease (intoxication)" must notify the following authority: Select one: a. Sanitary and epidemiological station b. Ministry of Health of Ukraine c. Medical unit of the plant d. Plant administration e. Trade union committee of the plant Feedback The correct answer is: Sanitary and epidemiological station Question 121 Not answered Mark 0.00 out of 1.00 Flag question Question text What methods of the collecting of the information is preferable for study of housing conditions of students of medical HIGH SCHOOL for a training period? Select one: a. Questioning b. Statistical c. Selecting of materials d. A method of the directed selection e. Interviewing
  • 71. Feedback The correct answer is: Questioning Question 122 Not answered Mark 0.00 out of 1.00 Flag question Question text The average body lenth of newborn boys is 50,9 cm at a sigma 1,66; and average mass - 3432 at a sigma 5,00. What criterion is necessary in order to compare degree of variability of these signs? Select one: a. Sigma b. Coefficient of association c. Amplitude d. Coefficient of variation e. Limit Feedback The correct answer is: Coefficient of variation Question 123 Not answered Mark 0.00 out of 1.00 Flag question Question text Prevalence of a disease in region N amounted 1156 occurences per 1000 of inhabitants. What of the mentioned indices characterizes the disease prevalence? Select one: a. Intensive b. Ratio c. Standardized
  • 72. d. Visual index e. Extensive Feedback The correct answer is: Intensive Question 124 Not answered Mark 0.00 out of 1.00 Flag question Question text While asessing the health status of graduates of a secondary school, the doctor found one of them to have grade 3 tonsillar hypertrophy, chronic rhinitis and vegetative-vascular dystonia. The organism functionality is reduced. This student belongs to the following health group: Select one: a. IV b. I c. V d. III e. II Feedback The correct answer is: III Question 125 Not answered Mark 0.00 out of 1.00 Flag question Question text The parameter of infantile mortality for the last year was - 16,3, in present year - 15,7. Name a kind of the diagram that can be used for a graphic representation of it:
  • 73. Select one: a. Radial b. Sector c. Linear d. Intrastylar e. Stylar Feedback The correct answer is: Stylar Question 126 Not answered Mark 0.00 out of 1.00 Flag question Question text In order to study impact of microclimate upon the human organism it is necessary to make systematic observation of air temperature over 3 days. Choose a device that will allow to make the most precise temperature records: Select one: a. August's psychrometer b. Alcohol thermometer c. Thermograph d. Assmann psychrometer e. Mercury thermometer Feedback The correct answer is: Thermograph Question 127 Not answered Mark 0.00 out of 1.00
  • 74. Flag question Question text A patient with high temperature came to a first-aid post in the evening. The fact of temporary disability was established. Indicate the order of examination in this case: Select one: a. The sick list for 3 days should be issued b. The sick list for up to 3 days should be issued c. Any document shouldn't be issued d. The sick list for 1 day should be issued e. The night duty doctor should issue a medical certificate, which will be subsequently used for issuing a sick list from the date of the previous day Feedback The correct answer is: The night duty doctor should issue a medical certificate, which will be subsequently used for issuing a sick list from the date of the previous day Question 128 Not answered Mark 0.00 out of 1.00 Flag question Question text A general practitioner visited a 2-year-old child and diagnosed him with measles. The child attends a nursery, has a 5-year-old sister. What document must be filled in for the effective antiepidemic measures in the given health locality? Select one: a. Infant's record (report form № 112/o) b. House call record (form № 031/o) c. Carer's leave certificate d. Sick leave e. Emergency notification on infectious disease (form № 058/o)
  • 75. Feedback The correct answer is: Emergency notification on infectious disease (form № 058/o) Question 129 Not answered Mark 0.00 out of 1.00 Flag question Question text Clinic of a research instutute for occupational diseases examined a worker who works at a concentration plant and diagnosed him with chronic dust bronchitis. The case is investigated by a commission including the representatives of: the plant, clinic, territorial SES, department of Social Insurance Fund, trade union. According to the "regulation on investigation of…", the commission should be headed by the representative of the following authority: Select one: a. Plant b. Trade union c. Social Insurance Fund d. Clinic e. Territorial SES Feedback The correct answer is: Territorial SES Question 130 Not answered Mark 0.00 out of 1.00 Flag question Question text A 5 tons milk batch was sampled. The lab analysis revealed: fat content 2%, specific density - 1,04 g/cm3, acidity - 210Т, reductase probe - weak-positive. What way is the product to be used in?
  • 76. Select one: a. Discard for animal feeding b. Sell but inform customers about milk quality c. Sell without limitations d. Do the product away e. Technical utilization Feedback The correct answer is: Sell but inform customers about milk quality Question 131 Not answered Mark 0.00 out of 1.00 Flag question Question text During a regular medical examination at a metallurgical plant 20% of workers were found overweight (body weight was 5-14% higher than normal), and had early signs of obesity (grade I-II) with Quetelet index from 26 to 30. What products share must be reduced in the diet of this group of people in the first place in order to normalize their body weight? Select one: a. Bakery products b. Milk and dairy products c. Vegetables d. Meat and fish products e. Fruit Feedback The correct answer is: Bakery products Question 132 Not answered Mark 0.00 out of 1.00
  • 77. Flag question Question text At a machine-building plant the casts are cleaned by means of abrasion machines that are a source of local vibration. What are the most efficient preventive measures for preventing harmful effect of vibration on workers' organisms? Select one: a. Preliminary and periodical medical examinations b. Warm hand baths c. Use of gloves that reduce vibration d. Giving sanitary instructions to the workers e. Hand massaging Feedback The correct answer is: Use of gloves that reduce vibration Question 133 Not answered Mark 0.00 out of 1.00 Flag question Question text A maternity hospital registered 616 live births, 1 stillbirth, 1 death on the 5th day of life over a 1 year period. What index allows the most precise estimation of this situation? Select one: a. Natality b. Crude mortality rate c. Natural increase d. Neonatal mortality e. Perinatal mortality
  • 78. Feedback The correct answer is: Perinatal mortality Question 134 Not answered Mark 0.00 out of 1.00 Flag question Question text Production areas of a greenhouse complex have the following microclimate parameters: air temperature - 42oC, humidity - 98%, air velocity - 0,05 mps, temperature of enclosing surfaces - 15oC. Characterize the microclimate of production areas: Select one: a. Uncomfortable b. Satisfactory c. Cooling d. Overheated e. Comfortable Feedback The correct answer is: Overheated Question 135 Not answered Mark 0.00 out of 1.00 Flag question Question text 25 children at the age of 2-3 years who don't attend any child welfare institutions should be observed by a district pediatrician within the current year. How many initial visits of this group of children should be planned? Select one: a. 200 b. 20
  • 79. c. 40 d. 50 e. 100 Feedback The correct answer is: 50 Question 136 Not answered Mark 0.00 out of 1.00 Flag question Question text Carpathian region is characterized by permanently high (over 80%) air humidity. In the cold season the population of this region feels very cold at moderately low temperatures. This is due to an increase in the heat transfer by: Select one: a. Radiation b. Conduction c. Emission d. Convection e. Evaporation Feedback The correct answer is: Convection Question 137 Not answered Mark 0.00 out of 1.00 Flag question Question text
  • 80. The institutions which take part in medical examinations can be prevention and treatment facilities, medical board of Ministry of Defense, medical board of Ministry of Home Affairs, medico-social expert commissions, forensic medical boards etc. What institutions are responsible for temporary disability examination? Select one: a. Prevention and treatment facilities b. Medical boards of the Ministry of Home Affairs c. Medico-social expert commissions d. Sanitary-and-prophylactic institutions e. Medical boards of the Ministry of Defense Feedback The correct answer is: Prevention and treatment facilities Question 138 Not answered Mark 0.00 out of 1.00 Flag question Question text Basing upon the data of laboratory assessment of sanitary state of soil in a certain territory, the soil was found to be low-contaminated according to the sanitary indicative value; contaminated according to the coli titer; low-contaminated according to the anaerobe titer (Cl. Perfringens). This is indicative of: Select one: a. Insufficient intensity of soil humification b. Insufficient insolation and aeration of soil c. Old fecal contamination d. Constant entry of organic protein contaminations e. Fresh fecal contamination Feedback The correct answer is: Fresh fecal contamination Question 139 Not answered
  • 81. Mark 0.00 out of 1.00 Flag question Question text Workers of a laboratory producing measuring devices (manometers, thermometers etc) complain about a mettalic taste in mouth, stomatitis, dyspepsia, sleep disturbance, unsteady walk, abrupt decrease in cardiac activity. These presentations must have been caused by the intoxication with the following substance: Select one: a. Tetraethyl lead b. Lead c. Manganese d. Toluol e. Mercury Feedback The correct answer is: Mercury Question 140 Not answered Mark 0.00 out of 1.00 Flag question Question text Study of morbidity with temporary disability among workers of a machine building plant revealed that average duration of a case was 20 days. What diseases influenced upon the index value? Select one: a. Acute b. Subacute c. Chronic d. Preexisting diseases e. Hard to determine
  • 82. Feedback The correct answer is: Chronic Question 141 Not answered Mark 0.00 out of 1.00 Flag question Question text A sample of milk was taken for testing from a 5 ton milk batch. Lab analysis showed the following: fat content 2%, specific density- 1,04 g/cm3, acidity 210C, reductase probe – weak positive. What way the product is to be used in? What would you advise? Select one: a. Sell without limitations b. Annihilate the product c. Sell but inform customers about milk quality d. Utilize technically e. Write the product off for animal feeding Feedback The correct answer is: Sell but inform customers about milk quality Question 142 Not answered Mark 0.00 out of 1.00 Flag question Question text It is determined that 30 of a 100 women with risk factor had preterm labor, and of a 100 women without risk factor 5 women had preterm labor. What method of statistic data processing should the doctor use in order to estimate reliability of differences between the compared groups? Select one:
  • 83. a. Relative numbers calculation b. Standardization method c. Average computing d. Correlation analysis e. Student's criterion calculation Feedback The correct answer is: Student's criterion calculation Question 143 Not answered Mark 0.00 out of 1.00 Flag question Question text At the radiological unit of a hospital gamma-devices of radiotherapy of "Agat" type and other closed sources of ionizing radiation are used for treatment of malignant neoplasms. What measures are to be taken to protect personnel during working with radioactive sources of such type? Select one: a. The increase of distance to the source and individual hygiene compliance b. Reduction of working time and screening of the source c. Screening of the source and the use of means of individual protection of respiration organs d. Systematical cleansing of surfaces from the radioactive contamination and shortening of working time e. Capsulation of devices and organization of room ventilation Feedback The correct answer is: Reduction of working time and screening of the source Question 144 Not answered Mark 0.00 out of 1.00
  • 84. Flag question Question text A worker who undergoes regular medical check-up for duodenal ulcer received a subsidized 24-day sanatorium voucher from his plant. The term of annual leave of a worker is 24 calender days, it will take 4 days more to get to the sanatorium and back home. What is the procedure of obtaining a 4- day sick-leave? Select one: a. Medical Expert Commission issues a 4-day health certificate b. The doctor in charge issues a 4-day sick list c. Medical Expert Commission issues a 4-day sick list d. Medical Expert Commission issues a 28-day sick list e. The doctor in charge issues a health certificate and sanatorium patient's file for 28 days Feedback The correct answer is: Medical Expert Commission issues a 4-day sick list Question 145 Not answered Mark 0.00 out of 1.00 Flag question Question text Sanitary-veterinary examination of a cow carcass revealed measle contamination (2-3 measles per 10 cm2). What tactics should be chosen in respect of this meat consumption? Select one: a. Meat should be disinfected by freezing b. The whole carcass should be technically disposed c. The carcass should be used for the production of canned meat d. Meat can be sold without any restrictions e. Meat should be disinfected by boiling
  • 85. Feedback The correct answer is: The whole carcass should be technically disposed Question 146 Not answered Mark 0.00 out of 1.00 Flag question Question text Maximum permissible concentration of carbon dioxide in the air is considered to be a sanitary index of air purity in a classroom. What concentration of carbon dioxide in the air is accepted as maximum permissible? Select one: a. 0,15% b. 0,1% c. 0,2% d. 0,3% e. 0,05% Feedback The correct answer is: 0,1% Question 147 Not answered Mark 0.00 out of 1.00 Flag question Question text Define the basic registration document at the profound study of a case rate with temporary lost labor ability at the industrial enterprise: Select one: a. The sick-leave certificate
  • 86. b. The inpatient medical record c. A card of the personal account of a case rate d. "The Report on reasons of a temporary lost labor ability" e. A ambulatory medical card Feedback The correct answer is: A card of the personal account of a case rate Question 148 Not answered Mark 0.00 out of 1.00 Flag question Question text Estimation of physical development of a child involved dynamometry and estimation of body weight and length, annual gain in body length, chest circumference, number of permanent teeth, secondary sexual characters, lung vital capacity. Which of the mentioned indices relates to the physiometric ones? Select one: a. Annual gain in body length b. Secondary sexual characters c. Body length and weight, chest circumference d. Lung vital capacity, dynamometry e. Number of permanent teeth Feedback The correct answer is: Lung vital capacity, dynamometry Question 149 Not answered Mark 0.00 out of 1.00 Flag question
  • 87. Question text A student analyzes noise level of cold-pressing process. What device should be applied for this hygienic study? Select one: a. Noise and vibration analyzer b. Pyranometer c. Sound tester d. Actinometer e. Noise analyzer Feedback The correct answer is: Noise and vibration analyzer Question 150 Not answered Mark 0.00 out of 1.00 Flag question Question text In the current year general practitioners of the municipal polyclinic have referred 11 patients with coronary artery disease to the in-patient hospital. In 3 cases the diagnosis wasn't confirmed. What managerial decision shoud be made in such case? Select one: a. Analysis of diagnostic examination quality b. Analysis of doctors' skill level c. Analysis of medical check-up quality d. Analysis of each case of diagnostic divergence e. Analysis of material and technical basisof the polyclinic Feedback The correct answer is: Analysis of each case of diagnostic divergence Question 151 Not answered Mark 0.00 out of 1.00
  • 88. Flag question Question text In order to reduce weed growth on agricultural land, some herbicides have been used for a long time. In terms of environmental stability these herbicides are rated as stable. Specify the most likely route of their entry into the human body: Select one: a. Soil-animals-humans b. Soil-microorganisms-humans c. Soil-protozoa-humans d. Soil-plants-humans e. Soil-insects-humans Feedback The correct answer is: Soil-plants-humans Question 152 Not answered Mark 0.00 out of 1.00 Flag question Question text During hygienic examination of a hospital it was established that the area for each bed in a double ward was: in the infectious department for children - 7 м2, in the infectious department for adults - 8 м2, in the burns department - 9 м2, in the radiological department - 10 м2, in the critical care department - 13 м2. In which wards the area for each bed doesn't correspond with hygienic requirements? Select one: a. In critical care wards b. In infectious wards for children c. In radiological wards d. In infectious wards for adults
  • 89. e. In burns wards Feedback The correct answer is: In burns wards Question 153 Not answered Mark 0.00 out of 1.00 Flag question Question text Evaluation results of sanitary and hygiene conditions in a 4-bed ward were as follows: ward area - 30 m2, height - 3,2 m, temperature - 20oC, humidity - 55%, air velocity - 0,1 m/s, window-to-floor area ratio - 1:5, daylight ratio - 0,6%, concentration of carbon dioxide in the air - 0,1%. Which of the given indicators does not meet hygienic requirements? Select one: a. Air velocity b. Daylight ratio c. Concentration of carbon dioxide in the air d. Window-to-floor area ratio e. Ward area Feedback The correct answer is: Daylight ratio Question 154 Not answered Mark 0.00 out of 1.00 Flag question Question text The objective of a statistical study was to find out the extent of seeking medical care by the population. For this purpose 300 residents of the area were interviewed. Information was collected
  • 90. by means of a special questionnaire. What method of collecting information was used by researchers? Select one: a. Doing extracts b. Immediate registration c. Anamnestic d. Immediate examination Feedback The correct answer is: Anamnestic Question 155 Not answered Mark 0.00 out of 1.00 Flag question Question text At's planned to construct multifield a new hospital in one of the cental city districts. What building type is the most appropriate in this case? Select one: a. Decentralized b. Mixed c. Blocked d. Centralized and blocked e. Centralized Feedback The correct answer is: Centralized and blocked Question 156 Not answered Mark 0.00 out of 1.00
  • 91. Flag question Question text It is planned to build a multi-disciplinary hospital with 500 beds in a town. Specify the location of a polyclinic within the medical centre: Select one: a. At the main entrance b. In the service zone c. It is not allowed to place the polyclinic within the centre territory d. In the centre of the territory near medical buildings e. In the garden and park area Feedback The correct answer is: At the main entrance Question 157 Not answered Mark 0.00 out of 1.00 Flag question Question text A patient who had eaten mushrooms in the morning was delivered to the infectious diseases hospital at night. The disease development was rapid. The patient presented with stomach pain, frequent diarrhea, intractable vomiting, burning thirst, headache and dizziness. He died on the third day. What mushrooms are most likely to have caused mycetismus? Select one: a. Sulfur-tufts b. Deadly amanita c. Russules d. Morels e. Fly agarics
  • 92. Feedback The correct answer is: Deadly amanita Question 158 Not answered Mark 0.00 out of 1.00 Flag question Question text A patient is on the sick leave for 4 months continuously from the date of injury. The treatment is going to last for 1-2 months more. Who has the right to extend the duration of medical certificate for this patient? Select one: a. Medical superintendent b. Medical advisory commission after inpatient treatment c. Medico-social expert commission d. Medical advisory commission after medico-social expert commission examination e. District doctor by agreement with a department chief Feedback The correct answer is: Medical advisory commission after medico-social expert commission examination Question 159 Not answered Mark 0.00 out of 1.00 Flag question Question text Examination of a 43-year-old man objectively revealed pallor of skin and mucous membranes, loss of tongue papillae, transverse striation of fingernails, cracks in the mouth corners, tachycardia. Blood test results: Hb- 90 g/l, anisocytosis, poikilocytosis. The most likely causative agent of this state is inadequate intake of:
  • 93. Select one: a. Zinc b. Iron c. Selene d. Magnesium e. Copper Feedback The correct answer is: Iron Question 160 Not answered Mark 0.00 out of 1.00 Flag question Question text A city somatic hospital with 300 beds consists of the main building which houses the therapeutic and surgical departments. Several separate buildings house the maternity, pediatric and radiologic departments that are connected to the main building by underground walkways and above-ground covered skybridges. Specify the building system of the hospital: Select one: a. Decentralized b. Combined c. Free d. Central-unit e. Centralized Feedback The correct answer is: Central-unit Question 161 Not answered Mark 0.00 out of 1.00
  • 94. Flag question Question text The correlation between the service record and eosinophil concentration in blood was studied in workers at dyeing shops of textile factories. What index will be the most informative for the analysis of this data? Select one: a. Sign index b. Fitting criterion c. Student's criterion d. Correlation factor e. Standardized index Feedback The correct answer is: Correlation factor Question 162 Not answered Mark 0.00 out of 1.00 Flag question Question text Indices that characterize population health include demographic indices. What environment is used for calculation of these indices? Select one: a. Number of hospitalized people b. Employment number c. Number of population being liable to preventive examination d. Population number e. Number of patients
  • 95. Feedback The correct answer is: Population number Question 163 Not answered Mark 0.00 out of 1.00 Flag question Question text A number of viable fetuses per 1000 women at the age between 15 and 44 is determined by: Select one: a. Reproductive level b. Obstetric rate c. Perinatal rate d. Birth rate e. Genital index Feedback The correct answer is: Genital index Question 164 Not answered Mark 0.00 out of 1.00 Flag question Question text It is required to analyze the level of daylight illumination in a ward of therapeutics unit. What device should be applied to estimate the level of daylight illumination? Select one: a. Psychrometer b. Katathermometer c. Actinometer
  • 96. d. Anemometer e. Illuminometer Feedback The correct answer is: Illuminometer Question 165 Not answered Mark 0.00 out of 1.00 Flag question Question text Clinical and statistical study was devoted to the effect of a new pharmacological medication upon the patients with coronary heart disease. What parametric criterion (coefficient) can be used for estimation of results validity? Select one: a. Sign criterion b. Student's coefficient (t) c. Kolmogorov-Smirnov's criterion d. Conformity coefficient e. Wilcoxon's t-criterion Feedback The correct answer is: Student's coefficient (t)